Сила ампера действующая на проводник с током i в магнитном поле индукцией в: Формула силы Ампера в физике

Содержание

Формула силы Ампера в физике

Содержание:

Определение и формула силы Ампера

Определение

Сила, действующая на проводник с током в магнитном поле, называется силой Ампера. Ее обозначения: $\bar{F}, \bar{F}_A$ . Сила Ампера векторная величина. Ее направление определяет правило левой руки: следует расположить ладонь левой руки так, чтобы силовые линии магнитного поля входили в нее. Вытянутые четыре пальца указывали направление силы тока. В таком случае отогнутый на большой палец укажет направление силы Ампера (рис.1).

Закон Ампера

Элементарная сила Ампера ($d\bar{F}_A$) определена законом (или формулой) Ампера:

$$d \bar{F}_{A}=I d \bar{l} \times \bar{B}(1)$$

где I – сила тока, $d \bar{l}$ – малый элемент длины проводника – это вектор, равный по модулю длине проводника, направленный в таком же направлении как вектор плотности тока, $\bar{B}$ – индукция магнитного поля, в которое помещен проводник с током.

Иначе эту формулу для силы Ампера записывают как:

$$d \bar{F}_{A}=\bar{j} \times \bar{B} d V(2)$$

где $\bar{j}$ – вектор плотности тока, dV – элемент объема проводника.

Модуль силы Ампера находят в соответствии с выражением:

$$d F=I \cdot B \cdot d l \cdot \sin \alpha(3)$$

где $\alpha$ – угол между векторами магнитной индукции и направление течения тока. Из выражения (3) очевидно, что сила Ампера максимальна в случае перпендикулярности линий магнитной индукции поля по отношению к проводнику с током.

Силы, действующие на проводники с током в магнитном поле

Из закона Ампера следует, что на проводник с током, равным I, действует сила равная:

$$\bar{F}_{A}=I \int_{l} d \bar{l} \times \bar{B}(4)$$

где $\bar{B}$ магнитная индукция, рассматриваемая в пределах малого кусочка проводника dl. Интегрирование в формуле (4) проводят по всей длине проводника (l). Из выражения (4) следует, что на замкнутый контур с током I, в однородном магнитном поле действует сила Ампера равная $\bar{F}_{A}=0(H)$

Сила Ампера, которая действует на элемент (dl) прямого проводника с током I

1, помещённый в магнитное поле, которое создает другой прямой проводник, параллельный первому с током I2, равна по модулю:

$$d F=\frac{\mu_{0}}{2 \pi} \frac{I_{1} I_{2}}{d} d l(5)$$

где d – расстояние между проводниками, $\mu_{0}=4 \pi \cdot 10^{7}$ Гн/м(или Н/А2 ) – магнитная постоянная. Проводники с токами одного направления притягиваются. Если направления токов в проводниках различны, то они отталкиваются. Для рассмотренных выше параллельных проводников бесконечной длины сила Амперана единицу длины может быть вычислена по формуле:

$$\frac{F}{l}=\frac{\mu_{0}}{2 \pi} \frac{I_{1} I_{2}}{d}$$

Формулу (6) в системе СИ применяют для получения количественного значения магнитной постоянной.

Единицы измерения силы Ампера

Основной единицей измерения силы Ампер (как и любой другой силы) в системе СИ является: [FA]=H

В СГС: [FA]=дин

Примеры решения задач

Пример

Задание. Прямой проводник длины l с током I находится в однородном магнитном поле B. На проводник действует сила F. Каков угол между направлением течения тока и вектором магнитной индукции?

Решение. На проводник с током, находящийся в магнитном поле действует сила Ампера, модуль которой для прямолинейного проводника с током расположенном в однородном поле можно представить как:

$$F=F_{A}=I B \operatorname{lsin} \alpha$$

где $\alpha$ – искомый угол. Следовательно:

$$\alpha=\arcsin \left(\frac{F}{I B l}\right)$$

Ответ. $\alpha=\arcsin \left(\frac{F}{I B l}\right)$

Слишком сложно?

Формула силы Ампера не по зубам? Тебе ответит эксперт через 10 минут!

Пример

Задание. Два тонких, длинных проводника с токами лежат в одной плоскости на расстоянии d друг от друга. Ширина правого проводника равна a. По проводникам текут токи I1 и I2 (рис.1). Какова, сила Ампера, действующая на проводники в расчете на единицу длины?

Решение. За основу решения задачи примем формулу элементарной силы Ампера:

$$d \bar{F}_{A}=I d \bar{l} \times \bar{B}(2.1)$$

Будем считать, что проводник с током I1 создает магнитное поле, а другой проводник в нем находится.Станем искать силу Ампера, действующую на проводник с током I2. Выделим в проводнике (2) маленький элемент dx (рис.1), который находится на расстоянии x от первого проводника. Магнитное поле, которое создает проводник 1 (магнитное поле бесконечного прямолинейного проводника с током) в точке нахождения элементаdxпо теореме о циркуляции можно найти как:

$$B \cdot 2 \pi x=\mu_{0} I_{1} \rightarrow B=\frac{\mu_{0} I_{1}}{2 \pi x}$$

Вектор магнитной индукции в точке нахождения элемента dx направлен перпендикулярно плоскости рисунка, следовательно, модуль элементарной силы Ампера, действующий на него можно представить как:

$$B \cdot 2 \pi x=\mu_{0} I_{1} \rightarrow B=\frac{\mu_{0} I_{1}}{2 \pi x}$$

где ток, который течет в элементе проводника dx, выразим как:

$$B \cdot 2 \pi x=\mu_{0} I_{1} \rightarrow B=\frac{\mu_{0} I_{1}}{2 \pi x}$$

Тогда выражение для dFA, учитывая (2.2) и (2.4) запишем как:

$$B \cdot 2 \pi x=\mu_{0} I_{1} \rightarrow B=\frac{\mu_{0} I_{1}}{2 \pi x}$$

где из рис.1 видно, что $a \leq x \leq a+b$, по условию задачи силу следует найти на единицу длины, значит $0 \leq l \leq 1$ . Для нахождения суммарной силы Ампера, действующей на проводник (2) возьмем двойной интеграл от выражения (2.{a+b} \frac{\mu_{0} I_{1}}{2 \pi x} \cdot \frac{I_{2}}{b} d x=\frac{\mu_{0} I_{1}}{2 \pi} \cdot \frac{I_{2}}{b} \ln \left|\frac{a+b}{a}\right|$$

Проводники действуют друг на друга с силами равными по модулю и так как токи направлены одинаково, то они притягиваются.

Ответ. $F_{A}=\frac{\mu_{0} I_{1}}{2 \pi} \cdot \frac{I_{2}}{b} \ln \left|\frac{a+b}{a}\right|$

Читать дальше: Формула силы выталкивания.

Магнитная сила Ампера

Возьмем прямой проводник, изготовленный из алюминия, и подвесим его на тонких и гибких проводах таким образом, чтобы он находился между полюсами подковообразного постоянного магнита как на рисунке (а). Если в проводнике пропустить ток, проводник отклонится от положения равновесия — рисунок (б). Причиной такого отклонения является сила, действующая на проводник с током со стороны магнитного поля. Доказал наличие этой силы и выяснил, от чего зависят ее значение и направление, французский физик, математик и химик Андре Мари Ампер. Именно поэтому это явление называют магнитной силой Ампера.

 

Сила Ампера — это сила, с которой магнитное поле действует на проводник с током.

Сила Ампера прямо пропорциональна силе тока в проводнике и длине активной части проводника (то есть части, которая расположена в магнитном поле). Сила Ампера увеличивается с увеличением индукции магнитного поля и зависит от того, под каким углом к ​​линиям магнитной индукции расположен проводник.

Значение силы Ампера (FA) вычисляют по формуле:

где В — магнитная индукция магнитного поля; I — сила тока в проводнике; l — длина активной части проводника; α — угол между направлением вектора магнитной индукции и направлением тока в проводнике.

Угол α — это угол между направлением вектора магнитной индукции и направлением тока в проводнике

Обратите внимание! Магнитное поле не будет действовать на проводник с током (FA= 0), если проводник расположен параллельно магнитным линиям поля (sin α = 0).

Определение
направления силы Ампера
по правилу левой руки

Чтобы определить направление силы Ампера, используют правило левой руки:

Если левую руку расположить так, чтобы линии магнитной индукции входили в ладонь, а четыре вытянутые пальцы указывали направление тока в проводнике, то отогнутый на 90 ° большой палец укажет направление силы Ампера.

На рисунке слева показано определения направления силы Ампера, действующая на проводник, расположенный в однородном магнитном поле. Давайте определим направление тока в проводнике, направление магнитной индукции и направление силы Ампера.

Получаем формулу для определения модуля магнитной индукции

Если проводник расположен перпендикулярно к линиям магнитной индукции (α = 90 °, sin α = 1), то поле действует на проводник с максимальной силой:

Отсюда получаем формулу для определения модуля магнитной индукции:

Обратите внимание! Значение магнитной индукции не зависит ни от силы тока в проводнике, ни от длины проводника, а зависит только от свойств магнитного поля.

Например, если уменьшить силу тока в проводнике, то изменится и сила Ампера, с которой магнитное поле действует на проводник, а вот значение магнитной индукции останется неизменным.

В СИ единица магнитной индукции — тесла (Тл), единица силы — ньютон (Н), силы тока — ампер (А), длины — метр (м), поэтому:

1Тл — это индукция такого однородного магнитного поля, которое действует с максимальной силой 1 Н на проводник длиной 1 м, в котором течет ток силой 1 А.

Проверочные задачи по теме: 

магнитное взаимодействие токов и сила Ампера

Задача 1. Докажите, что два параллельных проводника, в которых текут токи одного направления, притягиваются.

Анализ задачи:

Вокруг любого проводника с током существует магнитное поле, следовательно, каждый из двух проводников находится в магнитном поле другого. На первый проводник действует сила Ампера со стороны магнитного поля, созданного током во втором проводнике, и наоборот. Определив по правилу левой руки направления этих сил, выясним, как вести себя проводники.

Решение:

В ходе решения выполним объяснительные рисунки: изобразим проводники А и В, покажем направление тока в них и др.

Определим направление силы Ампера, действующая на проводник А, находящегося в магнитном поле проводника В.

1) С помощью правила буравчика определим направление линий магнитной индукции магнитного поля, созданного проводником В (рисунок слева). Выясняется, что у проводника А магнитные линии направлены к нам (отметка «•»).

2) Воспользовавшись правилом левой руки, определим направление силы Ампера, действующая на проводник А со стороны магнитного поля проводника В.

3) Приходим к выводу: проводник А привлекается к проводнику В.

Теперь найдем направление силы Ампера, действующая на проводник В, находится в магнитном поле проводника А.

1) Определим направление линий магнитной индукции магнитного поля, созданного проводником А (рисунок справа). Выясняется, что у проводника В магнитные линии направлены от нас (отметка «х»).

2) Определим направление силы Ампера, действующая на проводник В.

3) Приходим к выводу: проводник В привлекается к проводнику А.

Ответ: два параллельных проводника, в которых текут токи одного направления, действительно притягиваются.

Задача 2. Прямой проводник (стержень) длиной 0,1 м массой 40 г находится в горизонтальном однородном магнитном поле индукцией 0,5 Тл. Стержень расположен перпендикулярно магнитных линий поля). Ток какой силы и в каком направлении следует пропустить в стержне, чтобы он не давил на опору (завис в магнитном поле)?

Анализ задачи:

Стержень не будет давить на опору, если сила Ампера уравновесит силу тяжести. Это произойдет при следующих условиях:

  1. сила Ампера будет направлена ​​противоположно силе тяжести (то есть вертикально вверх)
  2. значение силы Ампера равна значению силы тяжести FA =  Fтяж

Направление тока определим, воспользовавшись правилом левой руки.

Решение:

Определим направление тока. Для этого расположим левую руку так, чтобы линии магнитного поля входили в ладонь, а отогнутый на 90 ° большой палец был направлен вертикально вверх. Четыре вытянутые пальцы укажут направление от нас. Итак, ток в проводнике следует направить от нас.

Учитываем, что FA =  FтяжFA= BIlsinα, где sin α = 1

; Fтяж = mg

Из последнего выражения найдем силу тока: I = mg/Bl

Проверим единицу, найдем значение искомой величины.

Ответ: I = 8 А; Ток в направлении от нас.

Подводим итоги

Силу, с которой магнитное поле действует на проводник с током, называют силой Ампера. Значение силы Ампера вычисляют по формуле: FA= BIlsinα, где B — индукция магнитного поля; I — сила тока в проводнике; l — длина активной части проводника; α — угол между направлением вектора магнитной индукции и направлением тока в проводнике.

Для определения направления магнитной силы Ампера используют правило левой руки: если левую руку расположить так, чтобы линии магнитного поля входили в ладонь, а четыре вытянутые пальцы указывали направление тока в проводнике, то отогнутый на 90 ° большой палец укажет направление силы Ампера.

7 «Б»

Урок

1/1

  Что изучает физика. Физические термины. Наблюдения и опыты. § 1 — 3, Л № 5, 12
2/2   Физические величины. Измерение физических величин. Погрешность и точность измерений § 4, 5, упр.1
3/3   Определение цены деления измерительного прибора § 4, 5
4/4   Физика и техника § 6,
    Первоначальные сведения о строении вещества  
5/1   Строение вещества. Молекулы § 7, 8
6/2   Определение размеров малых тел § 7, 8
7/3   Движение молекул. Диффузия в газах, жидкостях и твердых телах § 9,
8/4   Взаимодействие молекул

9/5

  Три состояния вещества § 11, 12
10/6   Повторение. Контрольная работа №1 «Первоначальные сведения о строении вещества» § 12
     

Сила, действующая на проводник с током в магнитном поле (сила Ампера). Сила, действующая на проводник с током в магнитном поле

Определение

Сила, действующая на проводник с током в магнитном поле, называется силой Ампера . Ее обозначения: . Сила Ампера векторная величина. Ее направление определяет правило левой руки: следует расположить ладонь левой руки так, чтобы силовые линии магнитного поля входили в нее. Вытянутые четыре пальца указывали направление силы тока. В таком случае отогнутый на большой палец укажет направление силы Ампера (рис.1).

Закон Ампера

Элементарная сила Ампера определена законом (или формулой) Ампера:

где I – сила тока, – малый элемент длины проводника – это вектор, равный по модулю длине проводника, направленный в таком же направлении как вектор плотности тока, – индукция магнитного поля, в которое помещен проводник с током.

Иначе эту формулу для силы Ампера записывают как:

где – вектор плотности тока, dV – элемент объема проводника.

Модуль силы Ампера находят в соответствии с выражением:

где – угол между векторами магнитной индукции и направление течения тока. Из выражения (3) очевидно, что сила Ампера максимальна в случае перпендикулярности линий магнитной индукции поля по отношению к проводнику с током.

Силы, действующие на проводники с током в магнитном поле

Из закона Ампера следует, что на проводник с током, равным I, действует сила равная:

где магнитная индукция, рассматриваемая в пределах малого кусочка проводника dl. Интегрирование в формуле (4) проводят по всей длине проводника (l). Из выражения (4) следует, что на замкнутый контур с током I, в однородном магнитном поле действует сила Ампера равная

Сила Ампера, которая действует на элемент (dl) прямого проводника с током I 1 , помещённый в магнитное поле, которое создает другой прямой проводник, параллельный первому с током I 2 , равна по модулю:

где d – расстояние между проводниками, Гн/м(или Н/А 2) – магнитная постоянная. Проводники с токами одного направления притягиваются. Если направления токов в проводниках различны, то они отталкиваются. Для рассмотренных выше параллельных проводников бесконечной длины сила Амперана единицу длины может быть вычислена по формуле:

Формулу (6) в системе СИ применяют для получения количественного значения магнитной постоянной.

Единицы измерения силы Ампера

Основной единицей измерения силы Ампер (как и любой другой силы) в системе СИ является: =H

В СГС: =дин

Примеры решения задач

Пример

Задание. Прямой проводник длины l с током I находится в однородном магнитном поле B. На проводник действует сила F. Каков угол между направлением течения тока и вектором магнитной индукции?

Решение. На проводник с током, находящийся в магнитном поле действует сила Ампера, модуль которой для прямолинейного проводника с током расположенном в однородном поле можно представить как:

где – искомый угол. Следовательно:

Ответ.

Пример

Задание. Два тонких, длинных проводника с токами лежат в одной плоскости на расстоянии d друг от друга. Ширина правого проводника равна a. По проводникам текут токи I 1 и I 2 (рис.1). Какова, сила Ампера, действующая на проводники в расчете на единицу длины?

Решение. За основу решения задачи примем формулу элементарной силы Ампера:

Будем считать, что проводник с током I 1 создает магнитное поле, а другой проводник в нем находится.Станем искать силу Ампера, действующую на проводник с током I 2 . Выделим в проводнике (2) маленький элемент dx (рис.1), который находится на расстоянии x от первого проводника. Магнитное поле, которое создает проводник 1 (магнитное поле бесконечного прямолинейного проводника с током) в точке нахождения элементаdxпо теореме о циркуляции можно найти как.

Закон Ампера показывает, с какой силой действует магнитное поле на помещенный в него проводник. Эту силу также называют силой Ампера .

Формулировка закона: сила, действующая на проводник с током, помещенный в однородное магнитное поле, пропорциональна длине проводника, вектору магнитной индукции, силе тока и синусу угла между вектором магнитной индукции и проводником .

Если размер проводника произволен, а поле неоднородно, то формула выглядит следующим образом:

Направление силы Ампера определяется по правилу левой руки.

Правило левой руки : если расположить левую руку так, чтобы перпендикулярная составляющая вектора магнитной индукции входила в ладонь, а четыре пальца были вытянуты по направлению тока в проводнике, то отставленный на 90 °большой палец, укажет направление силы Ампера.

МП движущего заряда. Действие МП на движущийся заряд. Сила Ампера, Лоренца.

Любой проводник с током создает в окружающем пространстве магнитное поле. При этом электрический же ток является упорядоченным движением электрических зарядов. Значит можно считать, что любой движущийся в вакууме или среде заряд порождает вокруг себя магнитное поле . В результате обобщения многочисленных опытных данных был установлен закон, который определяет поле В точечного заряда Q, движущегося с постоянной нерелятивистской скоростью v. Этот закон задается формулой

(1)

где r — радиус-вектор, который проведен от заряда Q к точке наблюдения М (рис. 1). Согласно (1), вектор В направлен перпендикулярно плоскости, в которой находятся векторы v и r: его направление совпадает с направлением поступательного движения правого винта при его вращении от v к r.

Рис.1

Модуль вектора магнитной индукции (1) находится по формуле

(2)

где α — угол между векторами v и r. Сопоставляя закон Био-Савара-Лапласа и (1), мы видим, что движущийся заряд по своим магнитным свойствам эквивалентен элементу тока: Idl = Qv

Действие МП на движущийся заряд.

Из опыта известно, что магнитное поле оказывает действие не только на проводники с током, но и на отдельные заряды, которые движутся в магнитном поле. Сила, которая действует на электрический заряд Q, движущийся в магнитном поле со скоростью v, называется силой Лоренца и задается выражением: F = Q где В — индукция магнитного поля, в котором заряд движется.

Чтобы определить направление силы Лоренца используем правило левой руки: если ладонь левой руки расположить так, чтобы в нее входил вектор В, а четыре вытянутых пальца направить вдоль вектора v (для Q>0 направления I и v совпадают, для Q На рис. 1 продемонстрирована взаимная ориентация векторов v, В (поле имеет направление на нас, на рисунке показано точками) и F для положительного заряда. Если заряд отрицательный, то сила действует в противоположном направлении.

Э.д.с. электромагнитной индукции в контуре пропорциональна скорости изменения магнитного потока Фm сквозь поверхность, ограниченную этим контуром:

где к — коэффициент пропорциональности. Данная э.д.с. не зависит от того, чем вызвано изменение магнитного потока — либо перемещением контура в постоянном магнитном поле, либо изменением самого поля.

Итак, направление индукционного тока определяется правилом Ленца: При всяком изменении магнитного потока сквозь поверхность, ограниченную замкнутым проводящим контуром, в последнем возникает индукционный ток такого направления, что его магнитное поле противодействует изменению магнитного потока.

Обобщением закона Фарадея и правила Ленца является закон Фарадея — Ленца: Электродвижущая сила электромагнитной индукции в замкнутом проводящем контуре численно равна и противоположна по знаку скорости изменения магнитного потока сквозь поверхность, ограниченную контуром:

Величину Ψ = ΣΦm называют потокосцеплением или полным магнитным потоком. Если поток, пронизывающий каждый из витков, одинаков (т.е. Ψ = NΦm), то в этом случае

Немецкий физик Г. Гельмгольц доказал, что закон Фарадея-Ленца является следствием закона сохранения энергии. Пусть замкнутый проводящий контур находится в неоднородном магнитном поле. Если в контуре течет ток I, то под действием сил Ампера незакрепленный контур придет в движение. Элементарная работа dA, совершаемая при перемещении контура за время dt, будет составлять

dA = IdФm,

где dФm — изменение магнитного потока сквозь площадь контура за время dt. Работа тока за время dt по преодолению электрического сопротивления R цепи равна I2Rdt. Полная работа источника тока за это время равна εIdt. По закону сохранения энергии работа источника тока затрачивается на две названные работы, т.е.

εIdt = IdФm + I2Rdt.

Разделив обе части равенства на Idt, получим

Следовательно, при изменении магнитного потока, сцепленного с контуром, в последнем возникает электродвижущая сила индукции

Электромагнитные колебания. Колебательной контур.

Электромагнитные колебания — это колебания таких величин, индуктивность, как сопротивление, ЭДС, заряд, сила тока.

Колебательный контур — это электрическая цепь, которая состоит из последовательно соединенных конденсатора, катушки и резистора. Изменение электрического заряда на обкладке кон- денсатора с течением времени описывается дифференциальным уравнением:

Электромагнитные волны и их свойства.

В колебательном контуре происходит процесс перехода электрической энергии конденсатора в энергию магнитного поля катушки и наоборот. Если в определенные моменты времени компенсировать потери энергии в контуре на сопротивление за счет внешнего источника, то получим незатухающие электрические колебания, которые через антенну могут быть излучены в окружающее пространство.

Процесс распространения электромагнитных колебаний, периодических изменений напряженностей электрического и магнитных полей, в окружающем пространстве называется электромагнитной волной.

Электромагнитные волны охватывают большой спектр длин волн от 105 до 10 м и по частотам от 104 до 1024 Гц. По названию электромагнитные волны разделяются на радиоволны, инфракрасное, видимое и ультрафиолетовое излучения, рентгеновские лучи и -излучение. В зависимости от длины волны или частоты свойства электромагнитных волн меняются, что является убедительным доказательством диалектико-материалистического закона перехода количества в новое качество.

Электромагнитное поле материальное и обладает энергией, количеством движения, массой, перемещается в пространстве: в вакууме со скоростью С, а в среде со скоростью: V= , где = 8,85 ;

Объемная плотность энергии электромагнитного поля. Практическое исполь-зование электромагнитных явлений весьма широкое. Это — системы и средства связи, радиовещания, телевидения, электронно-вычислительная техника, системы управления различного назна-чения, измерительные и медицинские приборы, бытовая электро- и радиоаппаратура и другие, т.е. то, без чего невозможно представить себе современное общество.

Как действует на здоровье людей мощное электромагнитное излучение, точных научных данных почти нет, есть только неподтвержденные гипотезы и, в общем-то, небезосновательные опасение, что все неестественное действует губительно. Доказано, что ультрафиолетовое, рентгеновское и -излучение большой интенсивности во многих случаях наносят реальный вред всему живому.

Геометрическая оптика. Законы ГО.

Геометрическая (лучевая) оптика использует идеализированное представление о световом луче — бесконечно тонком пучке света, распространяющемся прямолинейно в однородной изотропной среде, а также представления о точечном источнике излучения, равномерно светящем во все стороны. λ — длина световой волны, — характерный размер

предмета, находящегося на пути волны. Геометрическая оптика является предельным случаем волновой оптики и ее принципы выполняются при соблюдении условия:

h/D

В основе геометрической оптики лежит так же принцип независимости световых лучей: лучи при перемещении не возмущают друг друга. Поэтому перемещения лучей не мешают каждому из них распространяться независимо друг от друга.

Для многих практических задач оптики можно не учитывать волновые свойства света и считать распространение света прямолинейным. При этом картина сводится к рассмотрению геометрии хода световых лучей.

Основные законы геометрической оптики.

Перечислим основные законы оптики, следующие из опытных данных:

1) Прямолинейное распространение.

2) Закон независимости световых лучей, то есть два луча, пересекаясь, никак не мешают друг другу. Этот закон лучше согласуется с волновой теорией, так как частицы в принципе могли бы сталкиваться друг с другом.

3) Закон отражения. луч падающий, луч отраженный и перпендикуляр к поверхности раздела, восстановленный в точке падения луча, лежат в одной плоскости, называемой плоскостью падения; угол падения равен углу

Отражения.

4) Закон преломления света.

Закон преломления : луч падающий, луч преломленный и перпендикуляр к поверхности раздела, восстановленный из точки падения луча, лежат в одной плоскости — плоскости падения. Отношение синуса угла падения к синусу угла отражения равно отношению скоростей света в обеих средах.

Sin i1/ sin i2 = n2/n1 = n21

где — относительный показатель преломления второй среды относительно первой среды. n21

Если вещество 1 — пустота, вакуум, то n12 → n2 — абсолютный показатель преломления вещества 2. Можно легко показать, что n12 = n2 /n1 , в этом равенстве слева относительный показатель преломления двух веществ (например, 1 — воздух, 2 — стекло), а справа — отношение их абсолютных показателей преломления.

5) Закон обратимости света (его можно вывести из закона 4). Если направить свет в обратном направлении, он пройдёт по тому же пути.

Из закона 4) следует, что если n2 > n1 , то Sin i1 > Sin i2 . Пусть теперь у нас n2

Тогда можно понять, что при достижении некоторого значения этого угла (i1)пр окажется, что угол i2 окажется равным π /2 (луч 5). Тогда Sin i2 = 1 и n1 Sin (i1)пр = n2 . Итак Sin

В электрическом поле на поверхность проводника, а именно здесь расположены электрические заряды, действуют со стороны поля определённые силы. Поскольку напряжённость электростатического поля на поверхности проводника имеет только нормальную составляющую, сила, действующая на элемент площади поверхности проводника, является перпендикулярной этому элементу поверхности. Выражение для рассматриваемой силы, отнесённой к величине площади элемента поверхности проводника, имеет вид:

(1)

где — внешняя нормаль к поверхности проводника, — поверхностная плотность электрического заряда на поверхности проводника. Для заряженной тонкой сферической оболочки растягивающие усилия могут вызвать напряжения в материале оболочки, превышающие предел прочности.

Интересно, что подобные соотношения были предметом исследований таких классиков науки как Пуассон и Лаплас в самом начале XIX века. В соотношении (1) недоумение вызывает множитель 2 в знаменателе. Действительно, а почему правильный результат получается делением пополам выражения ? Рассмотрим один частный случай (рис.1): пусть проводящий шар радиуса содержит на своей боковой поверхности электрический заряд . Поверхностную плотность электрического заряда рассчитать легко: Введём сферическую систему координат (), элемент боковой поверхности шара определим как . Заряд элемента поверхности можно вычислить по зависимости: . Суммарный электрический заряд кольца радиуса и шириной определяется выражением: . Расстояние от плоскости рассматриваемого кольца до полюса сферы (боковая поверхность шара) равно . Известно решение задачи об определении составляющей вектора напряжённости электростатического поля на оси кольца (принцип суперпозиции) в точке наблюдения, отстоящей от плоскости кольца на расстояние :

Вычислим суммарное значение напряжённости электростатического поля, создаваемого поверхностными зарядами, исключая элементарный заряд в окрестности полюса сферы:

Вспомним, что около заряженной проводящей сферы напряжённость внешнего электростатического поля равна

Оказывается, сила, действующая на заряд элемента поверхности заряженного проводящего шара, в 2 раза меньше, чем сила, действующая на такой же заряд, расположенный вблизи боковой поверхности шара, но вне его.

Суммарная сила, действующая на проводник, равна

(5)

Помимо силы со стороны электростатического поля, проводник подвергается действию момента сил

(6)

где — радиус-вектор элемента поверхности dS проводника.

На практике часто оказывается более удобным силовое воздействие электростатического поля на проводник рассчитывать путем дифференцирования электрической энергии системы W. Сила, действующая на проводник, в соответствии с определением потенциальной энергии, равна

а величина проекции вектора момента сил на некоторую ось равна

где — угол поворота тела как целого вокруг рассматриваемой оси. Заметим, что приведенные выше формулы справедливы, если электрическая энергия W выражена через заряды проводников (источники поля!), а вычисление производных производится при постоянных значениях электрических зарядов.

Сила Ампера это та сила, с которой магнитное поле действует на проводник, с током помещённый в это поле. Величину этой силы можно определить с помощью закона Ампера. В этом законе определяется бесконечно малая сила для бесконечно малого участка проводника. Что дает возможность применять этот закон для проводников различной формы.

Формула 1 — Закон Ампера

B индукция магнитного поля, в котором находится проводник с током

I сила тока в проводнике

dl бесконечно малый элемент длинны проводника с током

альфа угол между индукцией внешнего магнитного поля и направлением тока в проводнике

Направление силы Ампера находится по правилу левой руки. Формулировка этого правила, звучит так. Когда левая рука расположена таким образом, что лини магнитной индукции внешнего поля входят в ладонь, а четыре вытянутых пальца указывают направление движения тока в проводнике, при этом отогнутый под прямым углом большой палец будет указывать направление силы, которая действует на элемент проводника.

Рисунок 1 — правило левой руки

Некоторые проблемы возникают, при использовании правила левой руки, в случае если угол между индукцией поля и током маленький. Трудно определить, где должна находиться открытая ладонь. Поэтому для простоты применения этого правила, можно ладонь располагать так, чтобы в нее входил не сам вектор магнитной индукции, а его модуль.

Из закона Ампера следует, что сила Ампера будет равна нулю, если угол между линией магнитной индукции поля и током будет равен нулю. То есть проводник будет располагаться вдоль такой линии. И сила Ампера будет иметь максимально возможное значение для этой системы, если угол будут составлять 90 градусов. То есть ток будет перпендикулярен линии магнитной индукции.

С помощью закона Ампера можно найти силу, действующую в системе из двух проводников. Представим себе два бесконечно длинных проводника, которые находятся на расстоянии друг от друга. По этим проводникам протекают токи. Силу, действующую со стороны поля создаваемого проводником с током номер один на проводник номер два можно представить в виде.

Формула 2 — Сила Ампера для двух параллельных проводников.

Сила, действующая со стороны проводника номер один на второй проводник, будет иметь такой же вид. При этом если токи в проводниках текут в одном направлении, то проводнику будут притягиваться. Если же в противоположных, то они будут отталкиваться. Возникает некоторое замешательство, ведь токи текут в одном направлении, так как же они могут притягиваться. Ведь одноименные полюса и заряды всегда отталкивались. Или Ампер решил, что не стоит подражать остальным и придумал что то новое.

На самом деле Ампер ничего не выдумывал, так как если задуматься то поля, создаваемые параллельными проводниками, направлены встречно друг другу. И почему они притягиваются, вопроса уже не возникает. Чтобы определить, в какую сторону направлено поле создаваемое проводником, можно воспользоваться правилом правого винта.

Рисунок 2 — Параллельные проводники с током

Используя параллельные проводники и выражение силы Ампера для них можно определить единицу в один Ампер. Если по бесконечно длинным параллельным проводникам, находящимся на расстоянии в один метр, текут одинаковые токи силой в одни ампер, то силы взаимодействия между ними будет составлять в 2*10-7 Ньютона, на каждый метр длинны. Используя эту зависимость, можно выразить чему будет равен один Ампер.

Данное видео рассказывает о том, как постоянное магнитное поле, созданное подковообразным магнитом, воздействует на проводник с током. Роль проводника с током в данном случае выполняет алюминиевый цилиндр. Этот цилиндр лежит на медных шинах, по которым к нему подводится электрический ток. Сила, воздействующая на проводник с током, находящемся в магнитном поле, называется силой Ампера. Направление действия силы Ампера определяется с помощью правила левой руки.

Французский физик Доминик Франсуа Араго (1786-1853) на заседании Парижской академии наук рассказал об опытах Эрстеда и повторил их. Араго предложил естественное, как всем казалось, объяснение магнитного действия электрического тока: проводник в результате протекания по нему электрического тока превращается в магнит. На демонстрации присутствовал другой академик, математик Андре Мари Ампер. Он предположил, что суть вновь открытого явления — в движении заряда, и решил сам провести необходимые измерения. Ампер был уверен, что замкнутые токи эквивалентны магнитам. 24 сентября 1820 г. он подключил к вольтову столбу две проволочные спирали, которые превратились в магниты.

Т.о. катушка с током создает такое же поле, что и полосовой магнит. Ампер создал прообраз электромагнита , обнаружив, что стальной брусок, помещенный внутрь спирали с током, намагничивается, многократно усиливая магнитное поле . Ампер предположил, что магнит представляет собой некоторую систему внутренних замкнутых токов и показал (и на основе опытов, и помощью расчетов), что малый круговой ток (виток) эквивалентен маленькому магнитику, расположенному в центре витка перпендикулярно его плоскости, т.о. всякий контур с током можно заменить магнитом бесконечно малой толщины.

Гипотеза Ампера, что внутри любого магнита существуют замкнутые токи, наз. гипотезой о молекулярных токах и легла в основу теории взаимодействия токов — электродинамики .

На проводник с током, находящийся в магнитном поле, действует сила, которая определяется только свойствами поля в том месте, где расположен проводник, и не зависит от того, какая система токов или постоянных магнитов создала поле. Магнитное поле оказывает на рамку с током ориентирующее действие. Следовательно, вращающий момент, испытываемый рамкой, есть результат действия сил на отдельные ее элементы.

Закон Ампера может быть использован для определения модуля вектора магнитной индукции. Модуль вектора индукции в данной точке однородного магнитного поля равен наибольшей силе, которая действует на помещенный в окрестности данной точки проводник единичной длины, по которому протекает ток в единицу силы тока: . Значение достигается при условии, что проводник расположен перпендикулярно к линиям индукции.

Закон Ампера применяется для определения силы взаимодействия двух токов.

Между двумя параллельно расположенными бесконечно длинными проводниками, по которым протекают постоянные токи, возникает сила взаимодействия. Проводники с одинаково направленными токами притягиваются, с противоположно направленными токами — отталкиваются.

Сила взаимодействия , приходящаяся на единицу длины каждого из параллельных проводников, пропорциональна величинам токов и и обратно пропорциональна расстоянию между R между ними. Такое взаимодействие проводников с параллельными токами объясняется правилом левой руки. Модуль силы, действующий на два бесконечных прямолинейных тока и , расстояние между которыми равно R .

Сила, действующая на проводник с током в магнитном поле

План решения задач

1. При расчете силы Ампера, действующей на проводник с током в магнитном поле, решение следует начать с рисунка, на котором нужно отразить форму проводника и направление вектора магнитной индукции поля, в котором находится проводник.

2. Необходимо иметь в виду, что формула силы Ампера справедлива только для прямого проводника с током длиной , который находится в однородном магнитном поле с индукцией . В случае неоднородного МП, а также для проводника криволинейной формы, проводник следует разделить на элементы тока и показать на рисунке векторы сил , действующих на элементы тока. Для этого необходимо выбрать два элемента тока, расположенных симметрично. Направление векторов определяем по правилу векторного произведения или по правилу левой руки: располагаем руку так, чтобы линии магнитной индукции входили в ладонь, четыре пальца направляем вдоль тока , тогда отогнутый большой палец покажет направление силы Сила, действующая на весь проводник, определяется как сумма векторов элементарных сил по всей длине проводника :

.

3. Свободный замкнутый контур с током (рамка или виток) устанавливается в магнитном поле так, чтобы его магнитный момент был сонаправлен с вектором магнитной индукции . При этом механический (вращающий) момент , а силы Ампера , действующие на элементы тока контура, растягивают его. Такое положение ( контура с током в однородном магнитном поле является состоянием устойчивого равновесия контура.

Задача 32. По трем параллельным прямым проводникам, находящимся на одинаковом расстоянии друг от друга (рис. 63 а) текут одинаковые токи В двух проводниках направления токов совпадают. Вычислите для каждого проводника силу, действующую на единицу длины проводника.

Дано Решение

Сначала рассмотрим взаимодействие двух проводников – первого и второго (рис. 63 б). На второй проводник с током действует магнитное поле с индукцией , созданное током в первом проводе (соответственно, и на первый проводник действует магнитное поле , созданное вторым проводом). Выберем на втором проводнике элемент тока , проведем линию магнитной индукции (это окружность радиусом ) и по касательной к ней направим вектор . Сила Ампера, действующая на выбранный элемент тока второго проводника со стороны МП первого тока

(1)

Модуль этой силы

, (2)

где угол между векторами и (линия магнитного поля расположена в плоскости, перпендикулярной проводу). Согласно формуле (2), сила, действующая со стороны первого провода на единицу длины второго провода:

(3)

В формуле (3) индукция МП, созданная прямым длинным проводом с током в точках на расстоянии от провода, определяется следующим выражением:

(4)

Направление силы определяем по правилу левой руки, располагая ладонь в плоскости рисунка: элемент тока притягивается к первому проводнику. По третьему закону Ньютона, на элемент тока первого проводника будет действовать сила , т. е. равная по модулю (см. формулу (3)) и противоположно направленная (см. рис. 63 б). Таким образом, параллельные токи одинакового направления притягиваются друг к другу. Изменим мысленно на рис. 63 б направление второго тока на противоположное (как ток ) и правило левой руки покажет, что сила, действующая на элемент тока , направлена вправо, т. е. параллельные токи противоположных направлений взаимно отталкиваются.

На каждый из проводников действуют магнитные поля двух других токов. Величину каждой силы парного взаимодействия -того и -того проводов запишем, подставляя индукцию магнитного поля, определяемую формулой (4) (в данной задаче ), в формулу (3):

. (5)

В соответствии с полученным выражением (5), величина силы парного взаимодействия на единицу длины одинакова для каждого проводника.

Результирующую силу, действующую на каждый проводник, находим с помощью принципа суперпозиции сил:

(6)

Покажем эти силы магнитного взаимодействия токов на рис. 63 в, учитывая, во-первых, взаимное направление токов, и во-вторых, равенство модулей всех сил парного взаимодействия . На рисунке заменим элементарную силу силой, действующей на весь i-тый провод со стороны -того тока, так как эти силы сонаправлены: .

Согласно формулам (6), сложим по два вектора сил, действующих на каждый проводник, геометрически: по правилу параллелограмма (треугольника) (см. рис. 63 в). Так как треугольники, имеющие сторонами векторы сил , равносторонние, то модули этих сил

(7)

Модуль силы найдем по теореме косинусов:

(8)

Силы, действующие на единицу длины провода, с учетом формулы (5), представятся выражениями, соответствующими формулам (7) и (8):

; (9)

(10)

Вычисляем силы: а) на единицу длины первого и второго провода:

.

б) на единицу длины третьего провода:

.

Задача 33. Квадратная проволочная рамка со стороной расположена в одной плоскости с длинным прямым поводом (рис. 64 а). Расстояние от провода до ближайшей стороны рамки . Ток в проводе , в рамке . Определите силы , действующие на каждую сторону рамки, и силу, действующую на всю рамку.

Дано Решение

Индукция магнитного поля, создаваемого длинным прямым проводом с током в точке, находящейся на расстоянии от провода, определяется следующей формулой:

. (1)

Величина уменьшается по мере увеличения расстояния , следовательно, это магнитное поле неоднородное. Направление вектора определяем по такому вращению буравчика, чтобы винт перемещался бы вдоль тока . В области, где находится рамка, вектор направлен перпендикулярно плоскости рамки «от нас» (рис. 64 б).

Найдем силу , действующую на сторону , суммируя бесконечно малые силы , действующие на элементы тока :

; (2)

(3)

По правилу левой руки определяем, что все векторы , перпендикулярные вектору магнитной индукции , лежат в плоскости рамки, а в этой плоскости они перпендикулярны стороне . Силы являются сонаправленными, причем, сторона притягивается к проводу, так как ток в ней одинакового направления с током в проводе (см. рис. 64 б). Модуль силы :

(4)

Здесь величина (в соответствии с формулой (1), в которой для стороны ) одинакова во всех точках МП, где находится сторона рамки . Тогда действующая на нее сила

(5)

Аналогичный расчет будет и для силы , действующей на сторону рамки , так как вдоль этой стороны величина также одинакова, но меньше, чем для стороны , так как расстояние от провода больше: . Соответственно и модуль силы :

(6)

Вектор также перпендикулярен стороне рамки ( ), но он направлен от провода с током : токи в проводе и в стороне противоположных направлений, поэтому они отталкиваются (см. рис. 64 б).

Силы , действующие на стороны и рамки с током, также перпендикулярны элементам тока и вектору магнитной индукции , в соответствии с векторным произведением в формуле (2), и направления их определяем также по правилу левой руки (см. рис. 64 б). Стороны рамки и расположены одинаково по отношению к проводу с током , магнитное поле которого действует на ток в рамке. Следовательно, модули этих сил одинаковы: .

Рассчитаем, например, силу , суммируя элементарные силы по длине стороны :

. (7)

Здесь величина не одинакова вдоль стороны , но уменьшается по мере удаления элемента тока от провода, согласно формуле (1). В подинтегральном выражении (7) заменим (см. рис. 64 б), чтобы перейти к одной переменной – расстоянию элемента тока от провода; пределы по этой переменной: , – соответствуют начальному и конечному элементам тока на стороне . Продолжим расчет силы

(8)

Вычислим модули сил, действующих на стороны рамки, по формулам (5), (6) и (8):

.

.

.

Найдем результирующую силу, действующую на рамку в целом, складывая векторы сил, действующих на стороны рамки:

(9)

Здесь , так как и вектор (см. рис. 64 б). Так как сила , то модуль результирующей силы

Направление вектора результирующей силы совпадает с направлением большего из векторов сил – с вектором .

Таким образом, в неоднородном магнитном поле на данную рамку с током действует сила в направлении градиента индукции МП: , который направлен в область более сильного МП. Силы растягивают рамку с током, что соответствует данному случаю , где – магнитный момент рамки с током.

Задача 34.На оси контура с током, магнитный момент которого , находится другой такой же контур. Магнитный момент второго контура перпендикулярен оси первого контура. Расстояние межу контурами , причем, размеры контуров малы по сравнению с расстоянием Определите механический момент , действующий на второй контур.

Дано Решение

Магнитный момент контура с током – это вектор , направленный по нормали к плоскости контура так, что направление вектора связано с направлением тока в контуре правилом буравчика (правого винта). Первый контур с током создает магнитное поле с индукцией . Величина в точках на оси кругового контура рассчитана в решении задачи 27:

, (1)

где – расстояние от точек контура до точки в МП, в которой определяется величина . Так как по условию задачи расстояние велико по сравнению с радиусом контура, то величина .

На второй контур с током в магнитном поле с индукцией действует механический (вращающий) момент , величина которого определяется следующей формулой:

. (2)

Так как размеры второго контура тоже малы, то величина несущественно изменяется вдоль плоскости второго контура. Поэтому примем ее равной , определяемой формулой (1), в которой . Согласно векторному произведению в формуле (2), вектор перпендикулярен плоскости, в которой лежат векторы и , т. е. он перпендикулярен плоскости рисунка (см. рис. 65). Этот механический момент будет стремиться повернуть второй контур до положения, в котором вектор (при этом величина обратится в нуль).

Модуль вращающего момента, согласно формуле (2),

, (3)

где – угол между векторами магнитного момента контура и индукцией магнитного поля . По условию задачи вектор , а последний создает магнитное поле , следовательно, вектор (см. рис. 65) и .

Подставляя величину магнитной индукции по формуле (1) в выражение (3), получаем следующую расчетную формулу:

. (4)

Вычисляем по формуле (4) механический момент, действующий на второй контур с током в магнитном поле, созданном первым контуром с током:

.

Задача 35.Два прямолинейных длинных параллельных проводника находятся на расстоянии друг от друга. По проводникам в одном направлении текут токи и . Какую работу (на единицу длины проводника) нужно совершить, чтобы раздвинуть эти проводники до расстояния ?

Дано Решение

Параллельные токи одинакового направления притягиваются друг к другу, т. е. второй проводник с током притягивается к первому силой Ампера . Чтобы его отодвинуть от первого проводника, нужно приложить внешнюю силу , незначительно превышающую силу притяжения проводников: . Работа этой внешней силы

(1)

Найдем силу Ампера – силу магнитного взаимодействия проводников с током, как силу, с которой магнитное поле первого проводника действует на ток во втором проводнике:

(2)

В уравнении (2) суммируются элементарные силы , действующие на элементы тока , расположенные по всей длине второго проводника с током. Направление сил определяем по правилу левой руки, размещая ладонь в плоскости рисунка (рис. 66), так как вектор магнитной индукции перпендикулярен плоскости рисунка (он направлен «к нам»). Силы , действующие на элементы тока , сонаправлены, поэтому можем складывать их модули:

(3)

Здесь , так как вектор ; – магнитная индукция поля, созданного прямым током , она определяется формулой

, (4)

где – расстояние от проводника с током до точки, в которой определяется индукция магнитного поля.

Подставим величину в подинтегральное выражение (3) и выполним интегрирование, отметив, что расстояние всех элементов тока второго проводника от первого одинаково, так как проводники параллельные:

(5)

Сила Ампера, действующая на единицу длины проводника, в соответствии с формулой (5), представится следующим выражением:

(6)

Согласно полученной формуле, эта сила уменьшается с увеличением расстояния между проводниками, т. е. имеем дело с работой переменной силы, которая определяется, как сумма элементарных работ, интегралом (1). Работу на единицу длины проводника найдем, подставляя силу по формуле (6) в подинтегральное выражение (1):

(7)

Вычислим работу, которую совершает внешняя сила при удалении от первого проводника с током второго проводника с током на единицу его длины, принимая, что магнитная проницаемость воздуха :

.

Задача 36.Тонкий проводник в виде полукольца радиусом находится в однородном магнитном поле с индукцией . Плоскость полукольца перпендикулярна линиям магнитной индукции, а подводящие провода расположены вдоль линий . По проводнику протекает ток . Определите силу , действующую на проводник.

Дано Решение

Выделим на полукольце элемент тока и определим направление действующей на него силы Ампера

(1)

Для этого используем правило левой руки, располагая ладонь в плоскости рисунка (рис. 67). Так как элементы тока кольцевого проводника имеют различную ориентацию, то векторы , перпендикулярные элементам тока , образуют «веер векторов» в плоскости полукольца. Для сложения таких векторов каждый элементарный вектор силы разложим на составляющие по осям :

(2)

Силу, действующую на весь проводник длины , находим, суммируя по всей длине полукольца векторы сил, действующих на элементы тока:

(3)

Покажем на рисунке вектор , действующий на элемент тока , расположенный симметрично элементу тока . По рисунку видно, что вектор , следовательно, они попарно компенсируются при суммировании и в результате этого Составляющие силы Ампера , действующие на все элементы тока, сонаправлены, поэтому векторное равенство (3) заменяем скалярным:

(4)

Здесь проекция силы (см. треугольник на рис. 67). Элементарная сила Ампера

, (5)

где – угол между векторами элемента тока и магнитной индукции ; по условию задачи , поэтому

Подставляя величину проекции силы в уравнение (4), перепишем его в следующем виде:

(6)

В подинтегральном выражении содержатся две переменные – элемент длины проводника и угол . Связь этих переменных находим из малого треугольника с гипотенузой (см. рис. 67): . Перейдем к переменной и запишем для нее пределы интегрирования. При сложении сил от всех элементов тока полукольца переменная изменяется от нуля (т. на рис. 67) до (т. на рис. 67), где – радиус полукольца. Тогда интеграл (6) принимает следующий вид:

(7)

Вычислим модуль силы Ампера, действующей в магнитном поле на полукольцо с током:

Вектор , а величина , следовательно, сила Ампера направлена вдоль оси (см. рис. 67).

Задача 37.Тонкий проводник в виде полукольца радиусом находится в однородном магнитном поле с индукцией . Вектор лежит в плоскости полукольца и перпендикулярен его диаметру (рис. 68). По проводнику течет ток . Определите силу , действующую на полукольцо.

Дано Решение

Сделаем чертеж (см. рис. 68), на котором покажем элемент тока , лежащий в плоскости рисунка. Сила Ампера, действующая на данный элемент тока, определяется по закону Ампера:

(1)

Силу, действующую на все элементы тока полукольца, найдем, суммируя элементарные силы:

. (2)

Согласно векторному произведению (1), сила перпендикулярна элементу тока и магнитной индукции . Так как оба вектора лежат в плоскости рисунка, то вектор силы перпендикулярен плоскости рисунка и направлен «к нам». Для всех элементов тока векторы сонаправлены, следовательно, и вектор силы , действующей на полукольцо, также направлен перпендикулярно плоскости рисунка.

Модуль этого вектора находим, используя формулы (1) и (2):

Узнать еще:

Сборник задач абитуриенту. МАГНЕТИЗМ. Сила Ампера. Индукция магнитного поля. Тема 22-1

          

МАГНЕТИЗМ. Сила Ампера. Индукция магнитного поля. Тема 22-1

22.1. На проводник длиной 0,5 м с током силой 20 A в однородном магнитном поле с индукцией 0,1 Тл действует сила 0,5 H. Какой угол (в градусах) составляет направление тока в проводнике с вектором магнитной индукции?

Ответ

22.2. Прямой проводник с током помещен в однородное магнитное поле перпендикулярно линиям индукции. Во сколько раз уменьшится сила, действующая на проводник со стороны магнитного поля, если его повернуть так, чтобы направление тока в проводнике составляло угол 30° с вектором индукции поля?

Ответ

22.3. Проводник с током помещен в однородное магнитное поле с индукцией B = 20 мТл. Определить силу, действующую на этот проводник, если его длина l = 0,1 м, сила тока I = 3,А, а угол между направлением тока и вектором B равен α = 45°.

Ответ

22.4. Проводник длиной 140 см согнули под прямым углом так, что одна из сторон угла равна 60 см, и поместили в однородное магнитное поле с индукцией 2 мТл обеими сторонами перпендикулярно линиям индукции. Какая сила (в мН) будет действовать на этот проводник, если по нему пропустить ток силой 10 A?

Ответ

22.5. Проводник длиной 110 см согнули под утлом 60° так, что одна из сторон угла равна 30 см, и поместили в однородное магнитное поле с индукцией 2 мТл обеими сторонами перпендикулярно линиям индукции. Какая сила (в мН) будет действовать на этот проводник, если по нему пропустить ток силой 10 A

Ответ

22.6. Определить направление силы взаимодействия тока с магнитным полем для каждого из случаев показанных на рисунке.

Ответ

22.7. Определить направление магнитного поля для каждого из случаев, показанных на рисунке. Проводник помещают перпендикулярно магнитному полю.

Ответ

22.8. По очень длинному вертикальному проводнику снизу вверх проходит ток силой 4 A. Определить положение точки вблизи проводника, в которой результирующий вектор магнитной индукции полей Земли и проводника имеет вертикальное направление. Горизонтальная составляющая индукции магнитного поля Земли 20 мкТл, провод расположен в северном полушарии.

Ответ

22.9. Два прямолинейных параллельных проводника с одинаковыми токами находятся на расстоянии 8 см друг от друга и притягиваются с силой 2,5 мН. Определить силу тока в проводниках, если их длина 250 см, а токи идут в одну сторону.

Ответ

Магнитное поле. Индукция магнитного поля. Сила, действующая на проводник с током в магнитном поле. Сила Ампера

Магнитное взаимодействие токов.
Магнитное поле. Индукция
магнитного поля.
Сила, действующая на проводник с
током в магнитном поле.
Сила Ампера (закон Ампера).

2. Магнитное поле. ЭМИ

Взаимодействие токов было открыто в 1820 году и изучено
Ампером, который исследовал поведение подвижных контуров
различной формы с током. Магнитное взаимодеймтвие
проводников отлично от электрического взаимодействия.
Электрическое взаимодействие зависит от наличия зарядов и от
их величины. Магнитное взаимодействие возникает только при
наличии токов и зависит от их величины. Проводники с
сонаправленными токами притягиваются, с противоположно
направленными токами — отталкиваются. Если заряженное тело
находится внутри замкнутой металлической оболочки,
электрического действия на него других зарядов не наблюдается,
тогда как магнитное действие на экранированный таким образом
проводник сохраняется.
Взаимодействие проводников с током обусловлено
возникновением вокруг них магнитного поля. Магнитное поле
возникает вокруг проводника с током всегда, даже если нет
другого проводника и отследить действие поля таким способом
нельзя.

3. Количественной характеристикой магнитного поля

служит специальная физическая величина — напряженность магнитного
поля H. С напряженностью связана также еще одна характеристика
магнитного поля — индукция В. Между ними существует соотношение:
B=mm0H,
m — магнитная проницаемость вещества.
Индукция и напряженность являются векторами.
Направление этих векторов подчиняется правилу правого буравчика:
направление магнитного поля совпадает с направлением движения конца
рукоядуи буравчика с правой нарезкой, движущегося поступательно в
направлении тока.
Сила, действующая на проводник с током в магнитном поле,
пропорциональна силе тока в проводнике I, магнитной индукции B, длине
проводника L и синусу угла между направлением тока в проводнике и
направлением вектора магнитной индукции a (Закон Ампера):
F=BLIsina.

4. Действие магнитного поля на движущийся заряд. Сила Лоренца

Поскольку на проводник с током в магнитном поле
действует сила, а ток есть направленное движение
заряженных частиц, можно сделать вывод, что на
каждый электрон действует некоторая сила (Сила
Лоренца):
F=evBsina,
где е — заряд электрона, v — его скорость, В магнитная индукция, a — угол между векторами v и В.
Правило определения направления силы Лоренца
такое же, как и для сила Ампера. Нужно иметь в виду,
что направление тока совпадает с направлением
движения положительных зарядов

5. Магнитные свойства вещества. Магнитная проницаемость. Ферромагнетизм

Некоторые вещества в магнитном поле намагничиваются, то есть сами
становятся источниками магнитного поля. Такие вещества называют
магнитиками. Механизм намагничивания следующий: в веществе есть
элементарные токи (замкнутые токи в пределах каждого атома), которые в
обычных условиях ориентированы хаотически, так что результирующий
магнитный момент равен нулю. Под действием внешнего магнитного поля эти
магнитные моменты ориентируются в одном направлении, и их векторная
сумма становится отлична от нуля.
Магнитное состояние вещества можно охарактеризовать с помощью
магнитного момента единицы объема. Эта величина называется вектор
намагничивания I.
Таким образом, для магнитика связь между векторами напряженности
магнитного поля и магнитной индукцией имеет вид:
B=H+4pI.
В общем случае, вектора I и H могут не совпадать. Это наблюдается для
некоторого класса веществ, называемых анизотропными магнитиками (в них в
них величина намагничения зависит еще и от направления внешнего поля в
веществе). Если же вещество является изотропным магнитиком, то вектора I и
H сонаправлены, то есть I=cH, где c — скалярная величина, называемая
магнитной воспиимчивостью.

6. Электромагнитная индукция. Магнитный поток. Закон электромагнитной индукции. Правило Ленца. Явление самоиндукции.

Индуктивность. Энергия магнитного поля
Известно, что проводник с током создает вокруг себя
магнитное поле. Верно и обратное: магнитное поле вызывает
появление электрических токов. Это явление получило
название электромагнитной индукции. Опыты показывают, что
причиной возникновения индукционного тока является
изменение магнитного поля. Это происходит в том случае,
если проводник пересекает магнитные силовые линии.
Полное количество линий магнитной индукции В, проходящих
через какую-либо поверхность, называют потоком магнитной
индукции Ф. В случае потока однородного магнитного поля
через плоский контур площадью S имеем:
Ф=BScosa,
Где a — угол между вектором В и направлением нормали к
плоскости контура. Если поле неоднородно, поток Ф
выражается интегралом:

8. Закон Ампера

Закон Ампера — закон взаимодействия постоянных токов.
Установлен Андре Мари Ампером в 1820. Из закона Ампера
следует, что параллельные проводники с токами, текущими в
одном направлении, притягиваются, а в противоположном —
отталкиваются. Законом Ампера называется также закон,
определяющий силу, с которой магнитное поле действует на
малый отрезок проводника с током.
Сила , с которой магнитное поле действует на элемент
объёма dV проводника с током плотности , находящегося в
магнитном поле с индукцией :
.
Если ток течёт по тонкому проводнику, то , где — «элемент
длины» проводника — вектор, по модулю равный dl и
совпадающий по направлению с током. Тогда предыдущее
равенство можно переписать следующим образом:

9. Два параллельных проводника

Магнитная сила на проводнике с током

Цели обучения

К концу этого раздела вы сможете:

  • Опишите влияние магнитной силы на проводник с током.
  • Рассчитайте магнитную силу на проводнике с током.

Поскольку заряды обычно не могут покинуть проводник, магнитная сила, действующая на заряды, движущиеся в проводнике, передается самому проводнику.

Рис. 1. Магнитное поле воздействует на провод с током в направлении, заданном правилом правой руки 1 (в том же направлении, что и на отдельные движущиеся заряды). Эта сила может быть достаточно большой, чтобы переместить провод, поскольку типичные токи состоят из очень большого количества движущихся зарядов.

Мы можем получить выражение для магнитной силы, действующей на ток, суммируя магнитные силы, действующие на отдельные заряды. (Силы складываются, потому что они в одном направлении.) Сила, действующая на отдельный заряд, движущийся со скоростью дрейфа v d , определяется как F = qv d B sin θ . Принимая B как однородный по длине провода l и ноль в другом месте, общая магнитная сила на проводе тогда будет F = ( qv d B sin θ ) ( N ) , где N — количество носителей заряда в отрезке провода длиной l .Теперь N = нВ, где n — количество носителей заряда на единицу объема, а В, — объем провода в поле. Заметим, что V = Al , где A — площадь поперечного сечения провода, тогда сила, действующая на провод, равна F = ( qv d B sin θ ) ( nAl ). Условия сбора,

[латекс] F = (nqAv _ {\ text {d}}) lB \ sin \ theta \\ [/ latex].

Потому что nqAv d = I (см. Ток),

[латекс] F = IlB \ sin \ theta \ [/ латекс]

— это уравнение для магнитной силы на длине l провода, по которому проходит ток I в однородном магнитном поле B , как показано на рисунке 2.Если разделить обе части этого выражения на l , мы обнаружим, что магнитная сила на единицу длины провода в однородном поле равна [латекс] \ frac {F} {l} = IB \ sin \ theta \\ [/ латекс]. Направление этой силы задается RHR-1 с большим пальцем в направлении тока I . Затем пальцами в направлении B перпендикуляр к ладони указывает в направлении F ​​, как на рисунке 2.

Рис. 2. Сила, действующая на токоведущий провод в магнитном поле, составляет F ​​ = IlB sin θ .Его направление задает RHR-1.

Пример 1. Расчет магнитной силы на проводе с током: сильное магнитное поле

Рассчитайте усилие на провод, показанное на рисунке 1, если B = 1,50 Тл, l = 5,00 см и I = 20,0 А.

Стратегия

Силу можно найти с данной информацией, используя [latex] F = IlB \ sin \ theta \\ [/ latex] и отметив, что угол θ между I и B равен 90º, так что sin θ = 1.

Решение

Ввод заданных значений в F ​​ = IlB sin θ дает

F ​​= IlB sin θ = (20,0 А) (0,0500 м) (1,50 Тл) (1).

Единицы измерения теслы: [латекс] 1 \ text {T} = \ frac {\ text {N}} {\ text {A} \ cdot \ text {m}} \\ [/ latex]; таким образом,

F ​​ = 1,50 Н.

Обсуждение

Это большое магнитное поле создает значительную силу на небольшой длине провода.

Магнитная сила на токоведущих проводниках используется для преобразования электрической энергии в работу. (Двигатели являются ярким примером — они используют проволочные петли и рассматриваются в следующем разделе.) Магнитогидродинамика (MHD) — это техническое название, данное умному приложению, в котором магнитная сила перекачивает жидкости без движущихся механических частей. (См. Рисунок 3.)

Рисунок 3. Магнитогидродинамика. Магнитная сила, действующая на ток, проходящий через эту жидкость, может использоваться в качестве немеханического насоса.

К трубке прикладывается сильное магнитное поле, и через жидкость проходит ток под прямым углом к ​​полю, в результате чего сила, действующая на жидкость, параллельна оси трубки, как показано. Отсутствие движущихся частей делает его привлекательным для перемещения горячего химически активного вещества, такого как жидкий натрий, используемый в некоторых ядерных реакторах. Экспериментальные искусственные сердца проходят испытания с использованием этого метода перекачивания крови, возможно, чтобы избежать неблагоприятного воздействия механических насосов.(Однако на клеточные мембраны влияют большие поля, необходимые в МГД, что задерживает его практическое применение у людей.) МГД-двигательная установка для атомных подводных лодок была предложена, поскольку она могла бы быть значительно тише, чем обычные гребные винты. Сдерживающая ценность атомных подводных лодок основана на их способности укрыться и пережить первый или второй ядерный удар. По мере того, как мы медленно разбираем наши арсеналы ядерного оружия, подводная ветка будет выведена из эксплуатации последней из-за этой способности (см.рисунок 4.) Существующие диски MHD тяжелые и неэффективные — требуется большая работа по развитию.

Рис. 4. Двигательная установка МГД на атомной подводной лодке может создавать значительно меньшую турбулентность, чем гребные винты, и позволять ей работать более бесшумно. Создание подводной лодки с бесшумным двигателем было инсценировано в книге и фильме Охота за красным октябрем .

Сводка раздела

  • Магнитная сила на токоведущих проводниках определяется выражением

    [латекс] F = IlB \ sin \ theta \ [/ латекс]

    , где I — ток, l — длина прямого проводника в однородном магнитном поле B , а θ — угол между I и B .Сила следует за RHR-1 большим пальцем в направлении I .

Концептуальные вопросы

  1. Нарисуйте схему ситуации на рисунке 1, показывающую направление электронов, переносящих ток, и используйте RHR-1, чтобы проверить направление силы на провод.
  2. Убедитесь, что направление силы в МГД-приводе, таком как на рисунке 3, не зависит от знака зарядов, переносящих ток через жидкость.
  3. Почему магнитогидродинамический привод лучше работает в океанской воде, чем в пресной? Кроме того, зачем нужны сверхпроводящие магниты?
  4. Что с большей вероятностью повлияет на показания компаса: переменный ток в холодильнике или постоянный ток при запуске автомобиля? Объяснять.

Задачи и упражнения

1. Каково направление магнитной силы, действующей на ток, в каждом из шести случаев на рисунке 5?

Рисунок 5.

2. Каково направление тока, который испытывает магнитную силу, показанную в каждом из трех случаев на рисунке 6, при условии, что ток течет перпендикулярно B ?

Рисунок 6.

3. Каково направление магнитного поля, которое создает магнитную силу, показанную на токах в каждом из трех случаев на рисунке 7, если предположить, что B перпендикулярно I ?

Рисунок 7.

4. (a) Какова сила на метр в разряде молнии на экваторе, несущем 20 000 А перпендикулярно полю Земли 3,00 × 10 −5 -T? (б) Каково направление силы, если ток идет прямо вверх, а направление поля Земли строго на север, параллельно земле?

5. (a) Линия электропередачи постоянного тока для системы легкорельсового транспорта передает ток 1000 А под углом 30º к полю Земли 5,00 × 10 −5 -T. Какая сила действует на 100-метровом участке этой линии? (b) Обсудите практические проблемы, которые это представляет, если таковые имеются.

6. Какая сила действует на воду в МГД-приводе, использующем трубку диаметром 25,0 см, если через трубку проходит ток 100 А, перпендикулярный магнитному полю 2,00 Тл? (Относительно небольшая величина этой силы указывает на необходимость очень больших токов и магнитных полей для создания практических МГД-приводов.)

7. Провод, по которому течет ток 30,0 А, проходит между полюсами сильного магнита, перпендикулярного его полю, и испытывает силу 2,16 Н на 4.00 см провода в поле. Какая средняя напряженность поля?

8. (a) Отрезок кабеля длиной 0,750 м, по которому подается ток к стартеру автомобиля, составляет угол 60º с полем Земли 5,50 × 10 −5 Тл. Каков ток, когда на провод действует сила 7,00 × 10 −3 Н? (b) Если вы пропустите провод между полюсами сильного подковообразного магнита, подвергнув его 5,00 см полю 1,75 Тл, какая сила будет приложена к этому отрезку провода?

9.(а) Каков угол между проводом, по которому течет ток 8,00 А, и полем 1,20 Тл, в котором он находится, если на 50,0 см провода действует магнитная сила 2,40 Н? б) Какая сила действует на проволоку, если ее повернуть на угол 90º с полем?

10. Сила, действующая на прямоугольную петлю из проволоки в магнитном поле на Рисунке 8, может использоваться для измерения напряженности поля. Поле однородное, плоскость петли перпендикулярна полю. а) Каково направление магнитной силы на петле? Обоснуйте утверждение, что силы на сторонах петли равны и противоположны, независимо от того, какая часть петли находится в поле, и не влияют на результирующую силу, действующую на петлю.(b) Если используется ток 5,00 А, какова сила на тесла на петле шириной 20,0 см?

Рис. 8. Прямоугольная петля из провода, по которой проходит ток, перпендикулярна магнитному полю. Поле однородно в показанной области и равно нулю за пределами этой области.

Избранные решения проблем и упражнения

1. (а) запад (слева)

(b) на стр.

(в) север (верх)

(d) нет силы

(д) восток (правый)

(е) юг (низ)

3.(a) на страницу

(б) запад (слева)

(c) вне страницы

5. (a) 2,50 Н (b) Это примерно полфунта силы на 100 м проволоки, что намного меньше веса самой проволоки. Поэтому особых опасений не вызывает.

7. 1,80 т

9. (а) 30º (б) 4.80 с.ш.

Магнитные поля, магнитные силы и проводники

Эффект Холла

Когда ток проходит по проводу, находящемуся под воздействием магнитного поля, в проводнике создается потенциал, поперечный току.

Цели обучения

Экспресс-напряжение Холла для металла, содержащего только один тип носителей заряда

Ключевые выводы

Ключевые моменты
  • Эффект Холла — это явление, при котором на электрическом проводнике возникает разность напряжений (называемая напряжением Холла), которая перпендикулярна электрическому току проводника, когда прикладывается магнитное поле, перпендикулярное току проводника.
  • Движущиеся заряды в проводе меняют траекторию в присутствии магнитного поля, «изгибаясь» к нему.Таким образом, эти заряды накапливаются на одной стороне материала. С другой стороны, остался избыток противоположного заряда. Таким образом создается электрический потенциал.
  • [латекс] \ text {V} _ \ text {H} = — \ frac {\ text {IB}} {\ text {net}} [/ latex] — это формула для напряжения Холла (В H ). Это фактор силы тока (I), магнитного поля (B), толщины проводящей пластины (t) и плотности носителей заряда (n) электронов-носителей.
Ключевые термины
  • элементарный заряд : электрический заряд одиночного протона.
  • поперечный : не касательный, поэтому между двумя пересекающимися объектами образуется невырожденный угол.

Эффект Холла — это явление, при котором на электрическом проводнике возникает разность напряжений (называемая напряжением Холла), поперечная электрическому току проводника, когда прикладывается магнитное поле, перпендикулярное току проводника.

Когда присутствует магнитное поле, которое не параллельно движению движущихся зарядов внутри проводника, на заряды действует сила Лоренца.В отсутствие такого поля заряды движутся примерно по прямой траектории, иногда сталкиваясь с примесями.

В присутствии магнитного поля с перпендикулярной составляющей пути, по которым проходят заряды, становятся искривленными, так что они накапливаются на одной стороне материала. С другой стороны, остается избыток противоположного заряда. Таким образом, электрический потенциал создается до тех пор, пока течет заряд. Это противодействует магнитной силе, в конечном итоге до точки компенсации, в результате чего поток электронов движется по прямому пути.

Эффект Холла для электронов : Сначала электроны притягиваются магнитной силой и движутся по изогнутой стрелке. В конце концов, когда электроны накапливаются в избытке на левой стороне и в дефиците на правой, создается электрическое поле ξy. Эта сила становится достаточно сильной, чтобы нейтрализовать магнитную силу, поэтому будущие электроны следуют по прямому (а не по кривой) пути.

Для металла, содержащего только один тип носителя заряда (электроны), напряжение Холла (V H ) можно рассчитать как коэффициент тока (I), магнитного поля (B) и толщины проводящей пластины (t). , и плотность носителей заряда (n) электронов носителей:

[латекс] \ text {V} _ \ text {H} = — \ frac {\ text {IB}} {\ text {net}} [/ latex]

В этой формуле e представляет собой элементарный заряд.

Коэффициент Холла (R H ) является характеристикой материала проводника и определяется как отношение индуцированного электрического поля (E y ) к произведению плотности тока (j x ) и приложенного магнитного поля. (В):

[латекс] \ text {R} _ \ text {H} = \ frac {\ text {E} _ \ text {y}} {\ text {j} _ \ text {xB}} = \ frac {\ text {V} _ \ text {Ht}} {\ text {IB}} = — \ frac {1} {\ text {ne}} [/ latex]

Эффект Холла — довольно распространенное явление в физике и проявляется не только в проводниках, но и в полупроводниках, ионизированных газах и, среди прочего, в квантовом спине.

Магнитная сила на проводнике, проводящем ток

Когда электрический провод подвергается воздействию магнита, ток в этом проводе испытывает силу — результат действия магнитного поля.

Цели обучения

Экспресс-уравнение, используемое для расчета магнитной силы электрического провода, находящегося в магнитном поле

Ключевые выводы

Ключевые моменты
  • Магнитная сила, действующая на ток, может быть найдена путем суммирования магнитной силы на каждом из отдельных зарядов, которые создают этот ток.
  • Для провода, подверженного воздействию магнитного поля, [латекс] \ text {F} = \ text {IlB} \ sin \ theta [/ latex] описывает взаимосвязь между магнитной силой (F), током (I) и длиной провода. (l), магнитное поле (B) и угол между полем и проводом (θ).
  • Направление магнитной силы может быть определено с помощью правила правой руки , как на рис [[17951]].
Ключевые термины
  • скорость дрейфа : средняя скорость свободных зарядов в проводнике.
  • магнитное поле : Состояние в пространстве вокруг магнита или электрического тока, в котором существует обнаруживаемая магнитная сила и где присутствуют два магнитных полюса.

Когда электрический провод подвергается воздействию магнита, на ток в этом проводе влияет магнитное поле. Эффект проявляется в виде силы. Выражение для магнитной силы, действующей на ток, можно найти, суммируя магнитную силу на каждом из множества отдельных зарядов, составляющих ток.Поскольку все они движутся в одном направлении, силы могут складываться.

Правило правой руки : Используется для определения направления магнитной силы.

Сила (F), которую магнитное поле (B) оказывает на отдельный заряд (q), движущийся со скоростью дрейфа v d , составляет:

[латекс] \ text {F} = \ text {qv} _ \ text {dB} \ sin \ theta [/ latex]

В этом случае θ представляет собой угол между магнитным полем и проводом (магнитная сила обычно рассчитывается как перекрестное произведение).Если B является постоянным по всему проводу и 0 в другом месте, то для провода с N носителями заряда на его общей длине l общая магнитная сила на проводе составляет:

[латекс] \ text {F} = \ text {Nqv} _ \ text {dB} \ sin \ theta [/ latex].

При условии, что N = nV, где n — количество носителей заряда в единице объема, а V — объем провода, и что этот объем рассчитывается как произведение площади круглого поперечного сечения A и длины (V = Al) , дает уравнение:

[латекс] \ text {F} = (\ text {nqAv} _ \ text {d}) \ text {lB} \ sin \ theta [/ latex].

Слагаемые в скобках равны току (I), поэтому уравнение можно переписать как:

[латекс] \ text {F} = \ text {IlB} \ sin \ theta [/ latex]

Направление магнитной силы может быть определено с помощью правила для правой руки , продемонстрированного в. Большой палец указывает в направлении тока, а четыре других пальца параллельны магнитному полю. Сгибание пальцев показывает направление магнитной силы.

Момент на токовой петле: прямоугольный и общий

Токоведущий контур, подверженный воздействию магнитного поля, испытывает крутящий момент, который может использоваться для питания двигателя.

Цели обучения

Определите общее предложение крутящего момента на петле любой формы

Ключевые выводы

Ключевые моменты
  • [латекс] \ tau = \ text {NIAB} \ sin \ theta [/ latex] можно использовать для расчета крутящего момента ([latex] \ tau [/ latex]) петли из N витков и площади, по которой проходит ток I чувствует себя в магнитном поле B.
  • Хотя силы, действующие на петлю, равны и противоположны, они обе действуют, вращая петлю в одном направлении.
  • Испытываемый крутящий момент не зависит от формы петли. Важна площадь петли.
Ключевые термины
  • крутящий момент : вращательное или скручивающее действие силы; (Единица СИ ньютон-метр или Нм; британская единица измерения фут-фунт или фут-фунт)

Когда ток проходит по петле, которая подвергается воздействию магнитного поля, это поле оказывает крутящий момент на петлю. Этот принцип обычно используется в двигателях, в которых контур соединен с валом, который вращается под действием крутящего момента.Таким образом, электрическая энергия тока преобразуется в механическую энергию при вращении петли и вала, и эта механическая энергия затем используется для питания другого устройства.

Крутящий момент на токовой петле : электрическая энергия тока преобразуется в механическую энергию при вращении петли и вала, и эта механическая энергия затем используется для питания другого устройства.

В этой модели северный и южный полюса магнитов обозначены буквами N и S соответственно. В центре — прямоугольная проволочная петля длиной l и шириной w, по которой проходит ток I.Воздействие магнитного поля B на токоведущий провод вызывает крутящий момент τ.

Чтобы понять крутящий момент, мы должны проанализировать силы, действующие на каждый сегмент контура. Предполагая постоянное магнитное поле, мы можем заключить, что силы в верхней и нижней частях петли равны по величине и противоположны по направлению, и, таким образом, не создают результирующей силы. Между прочим, эти силы вертикальны и, следовательно, параллельны валу.

Однако, как показано (a) на рисунке ниже, равные, но противоположные силы создают крутящий момент, действующий по часовой стрелке.

Изменяющийся крутящий момент на заряженном контуре в магнитном поле : Максимальный крутящий момент возникает в (b), когда равен 90 градусам. Минимальный крутящий момент равен 0 и встречается в (c), когда θ составляет 0 градусов. Когда контур вращается после = 0, крутящий момент меняется на противоположное (d).

Учитывая, что крутящий момент рассчитывается по уравнению:

[латекс] \ tau = \ text {rF} \ sin \ theta [/ latex]

где F — сила, действующая на вращающийся объект, r — расстояние от точки поворота, к которой приложена сила, а θ — угол между r и F, мы можем использовать сумму двух крутящих моментов (силы действуют по обе стороны от петли), чтобы найти общий крутящий момент:

[латекс] \ tau = \ frac {\ text {w}} {2} \ text {F} \ sin \ theta + \ frac {\ text {w}} {2} \ text {F} \ sin \ theta = \ text {wF} \ sin \ theta [/ latex]

Обратите внимание, что r равно w / 2, как показано на рисунке.

Чтобы найти крутящий момент, мы все равно должны найти F из магнитного поля B относительно тока I. Прямоугольник имеет длину l, поэтому F = IlB. Замена F на IlB в уравнении крутящего момента дает:

[латекс] \ tau = \ text {wIlB} \ sin \ theta [/ latex]

Обратите внимание, что произведение w и l включено в это уравнение; эти термины можно заменить площадью (A) прямоугольника. Если используется проволока другой формы, ее площадь можно вставить в уравнение независимо от формы (круглой, квадратной или другой).

Также обратите внимание, что это уравнение крутящего момента рассчитано на один оборот. Крутящий момент увеличивается пропорционально количеству оборотов (Н). Таким образом, общее уравнение для крутящего момента на петле любой формы, из N витков, каждая из областей A, несущая ток I и подверженная воздействию магнитного поля B, является величиной, которая колеблется при вращении петли, и может быть вычислена по формуле:

[латекс] \ tau = \ text {NIAB} \ sin \ theta [/ latex]

Закон Ампера: Магнитное поле из-за длинного прямого провода

Ток, протекающий по проводу, создает магнитное поле, которое можно рассчитать по закону Био-Савара.

Цели обучения

Выразите взаимосвязь между силой магнитного поля и током, протекающим через провод, в форме уравнения

Ключевые выводы

Ключевые моменты
  • Закон Ампера гласит, что для замкнутой кривой длиной C магнитное поле (B) связано с током (I C ): [латекс] \ oint_ \ text {C} {\ text {Bd} \ ell = \ mu _0 \ text {I} _ \ text {C}} [/ latex]. В этом уравнении dl представляет собой разницу длины провода в изогнутом проводе, а μ 0 — проницаемость свободного пространства.3} [/ латекс]. В этом уравнении парциальное магнитное поле (дБ) выражается как функция тока для бесконечно малого отрезка провода (dl) в точке на расстоянии r от проводника.
  • После интегрирования направление магнитного поля в соответствии с законом Био-Савара может быть определено с помощью правила правой руки.
Ключевые термины
  • электрическое поле : область пространства вокруг заряженной частицы или между двумя напряжениями; он воздействует на заряженные объекты поблизости.
  • магнитное поле : Состояние в пространстве вокруг магнита или электрического тока, в котором существует обнаруживаемая магнитная сила и где присутствуют два магнитных полюса.

Ток, протекающий по проводу, создает как электрическое, так и магнитное поле. Для замкнутой кривой длиной C магнитное поле (B) связано с током (I C ), как в Законе Ампера, выраженном математически как:

[латекс] \ oint_ \ text {C} {\ text {Bd} \ ell = \ mu _0 \ text {I} _ \ text {C}} [/ latex]

Направление магнитного поля : Направление магнитного поля можно определить по правилу правой руки. 3} [/ latex].2}} [/ латекс].

Это соотношение сохраняется для постоянного тока в прямом проводе, в котором магнитное поле в точке, обусловленное всеми токовыми элементами, составляющими прямой провод, одинаково. Как показано, направление магнитного поля может быть определено с помощью правила для правой руки — когда большой палец направлен в направлении тока, изгиб пальцев указывает направление магнитного поля вокруг прямого провода.

Магнитная сила между двумя параллельными проводниками

Параллельные провода, по которым проходит ток, создают значительные магнитные поля, которые, в свою очередь, создают значительные силы на токи.

Цели обучения

Выразите магнитную силу, ощущаемую парой проводов, в форме уравнения

Ключевые выводы

Ключевые моменты
  • Поле (B 1 ), создаваемое этим током (I 1 ) из провода, можно рассчитать как функцию тока и расстояния между проводами (r): [latex] \ text {B} _1 = \ frac {\ mu_0 \ text {I} _1} {2 \ pi \ text {r}} [/ latex] μ 0 — постоянная величина.
  • [латекс] \ text {F} = \ text {IlB} \ sin \ theta [/ latex] описывает магнитную силу, ощущаемую парой проводов.Если они параллельны, уравнение упрощается, так как функция синуса равна 1.
  • Сила, ощущаемая между двумя параллельными проводящими проводами, используется для определения ампера — стандартной единицы силы тока.
Ключевые термины
  • ампер : единица электрического тока; стандартная базовая единица в Международной системе единиц. Аббревиатура: amp. Символ: A.
  • ток : временная скорость протекания электрического заряда.
  • магнитное поле : Состояние в пространстве вокруг магнита или электрического тока, в котором существует обнаруживаемая магнитная сила и где присутствуют два магнитных полюса.

Параллельные провода, по которым проходит ток, создают значительные магнитные поля, которые, в свою очередь, создают значительные силы на токи. Сила, ощущаемая между проводами, используется для определения стандартной единицы тока, известной как амфера.

In поле (B 1 ), которое создает I 1 , может быть вычислено как функция тока и расстояния между проводами (r):

Магнитные поля и сила, создаваемые параллельными токоведущими проводами. : Токи I1 и I2 текут в одном направлении, разделенные расстоянием r.

[латекс] \ text {B} _1 = \ frac {\ mu_0 \ text {I} _1} {2 \ pi \ text {r}} [/ latex]

Поле B 1 оказывает давление на провод, содержащий I 2 . На рисунке эта сила обозначена как F 2 .

Сила F 2 , действующая на провод 2, может быть рассчитана как:

[латекс] \ text {F} _2 = \ text {I} _2 \ text {lB} _1 \ sin \ theta [/ latex]

Учитывая, что поле однородно вдоль и перпендикулярно проводу 2, sin θ = sin 90 derees = 1. Таким образом, сила упрощается до: F 2 = I 2 lB 1

Согласно Третьему закону Ньютона (F 1 = -F 2 ) силы на двух проводах будут равны по величине и противоположны по направлению, поэтому просто мы можем использовать F вместо F 2 .Учитывая, что провода часто бывают очень длинными, часто бывает удобно найти силу на единицу длины. Преобразуя предыдущее уравнение и используя определение B 1 , получаем:

[латекс] \ frac {\ text {F}} {\ text {l}} = \ frac {\ mu_0 \ text {I} _1 \ text {I} _2} {2 \ pi \ text {r}} [ / латекс]

Если токи в одном направлении, сила притягивает провода. Если токи идут в противоположных направлениях, сила отталкивает провода.

Сила между токоведущими проводами используется как часть рабочего определения силы тока.{-7} \ text {N} / \ text {m} [/ latex]

Последние единицы получены при замене Т на 1Н / (А × м).

Между прочим, это значение является основой рабочего определения ампера. Это означает, что один ампер тока через два бесконечно длинных параллельных проводника (разделенных одним метром в пустом пространстве и без каких-либо других магнитных полей) вызывает силу 2 × 10 -7 Н / м на каждый провод.

11,5: Магнитная сила на проводнике с током

Движущиеся заряды испытывают силу в магнитном поле.Если эти движущиеся заряды находятся в проводе, то есть если по проводу проходит ток, на провод также должна действовать сила. Однако, прежде чем обсуждать силу, действующую на ток со стороны магнитного поля, мы сначала исследуем магнитное поле, создаваемое электрическим током. Здесь мы изучаем два отдельных эффекта, которые тесно взаимодействуют: провод с током создает магнитное поле, а магнитное поле оказывает силу на провод с током.

Магнитные поля, создаваемые электрическим током

Обсуждая исторические открытия в области магнетизма, мы упомянули открытие Эрстеда о том, что провод, по которому проходит электрический ток, вызывает отклонение расположенного поблизости компаса.Было установлено, что электрические токи создают магнитные поля. (Эта связь между электричеством и магнетизмом более подробно обсуждается в Источниках магнитных полей.)

Стрелка компаса рядом с проволокой испытывает силу, которая выравнивает касательную иглы к окружности вокруг проволоки. Следовательно, токоведущий провод создает кольцевые петли магнитного поля. Чтобы определить направление магнитного поля, создаваемого проводом, мы используем второе правило правой руки.В RHR-2 ваш большой палец указывает в направлении тока, в то время как ваши пальцы охватывают провод, указывая в направлении создаваемого магнитного поля (рисунок \ (\ PageIndex {1} \)). Если магнитное поле попадало на вас или выходило за пределы страницы, мы обозначаем это точкой. Если магнитное поле попадало на страницу, мы обозначаем это знаком ×.

Эти символы получены с учетом векторной стрелки: стрелка, направленная на вас, с вашей точки зрения будет выглядеть как точка или кончик стрелки.Стрелка, направленная от вас, с вашей точки зрения будет выглядеть как крест или знак ×. Составной эскиз магнитных кругов показан на рисунке \ (\ PageIndex {1} \), где показано, что напряженность поля уменьшается по мере удаления от провода петлями, которые расположены дальше друг от друга.

Рисунок \ (\ PageIndex {1} \): (a) Когда провод находится в плоскости бумаги, поле перпендикулярно бумаге. Обратите внимание на символы, используемые для поля, указывающего внутрь (например, хвоста стрелки), и поля, указывающего наружу (например, кончика стрелки).(б) Длинный и прямой провод создает поле с силовыми линиями магнитного поля, образующими кольцевые петли.

Расчет магнитной силы

Электрический ток — это упорядоченное движение заряда. Следовательно, провод с током в магнитном поле должен испытывать силу, создаваемую этим полем. Чтобы исследовать эту силу, давайте рассмотрим бесконечно малое сечение провода, как показано на рисунке \ (\ PageIndex {3} \). Длина и площадь поперечного сечения секции равны дл и A соответственно, поэтому ее объем равен \ (V = A \ cdot dl \).Проволока сформирована из материала, который содержит n носителей заряда в единице объема, поэтому количество носителей заряда в секции равно \ (nA \ cdot dl \). Если носители заряда движутся со скоростью дрейфа \ (\ vec {v} _d \), ток I в проводе равен (от тока и сопротивления)

\ [I = neAv_d. \]

Магнитная сила на любом отдельном носителе заряда равна \ (e \ vec {v} _d \ times \ vec {B} \), поэтому общая магнитная сила \ (d \ vec {F} \) на \ (nA \ cdot дл \) носителей заряда в сечении провода

\ [d \ vec {F} = (nA \ cdot dl) e \ vec {v} _d \ times \ vec {B}.\]

Мы можем определить dl как вектор длиной dl , указывающий вдоль \ (\ vec {v} _d \), что позволяет нам переписать это уравнение как

\ [d \ vec {F} = neAv_dd \ vec {l} \ times \ vec {B}, \] или

\ [d \ vec {F} = Id \ vec {l} \ times \ vec {B}. \ label {11.12} \]

Это сила магнитного поля на отрезке провода. Обратите внимание, что на самом деле это результирующая сила, действующая со стороны поля на сами носители заряда. Направление этой силы задается RHR-1, где вы указываете пальцами в направлении тока и сгибаете их к полю.Затем ваш большой палец указывает в направлении силы.

Рисунок \ (\ PageIndex {2} \): бесконечно малое сечение токоведущего провода в магнитном поле.

Чтобы определить магнитную силу \ (\ vec {F} \) на проводе произвольной длины и формы, мы должны интегрировать уравнение \ ref {11.12} по всему проводу. Если сечение провода прямое, а B однородный, дифференциалы уравнения становятся абсолютными величинами, что дает нам

\ [\ vec {F} = I \ vec {l} \ times \ vec {B}.\]

Это сила, действующая на прямой провод с током в однородном магнитном поле.

Пример \ (\ PageIndex {1} \): уравновешивание гравитационных и магнитных сил на проводе с током

Провод длиной 50 см и массой 10 г подвешен в горизонтальной плоскости с помощью пары гибких проводов (рисунок \ (\ PageIndex {3} \)). Затем на проволоку действует постоянное магнитное поле величиной 0,50 Тл, которое направлено, как показано. Каковы величина и направление тока в проводе, необходимые для снятия напряжения в опорных выводах?

Рисунок \ (\ PageIndex {3} \): (a) Проволока, подвешенная в магнитном поле.(б) Схема свободного тела для проволоки.

Стратегия

Из диаграммы свободного тела на рисунке, натяжения в опорных выводах стремятся к нулю, когда гравитационная и магнитная силы уравновешивают друг друга. Используя RHR-1, мы обнаруживаем, что магнитная сила направлена ​​вверх. Затем мы можем определить ток I , приравняв две силы.

Решение

Приравняйте две силы веса и магнитной силы к проводу:

\ [мг = IlB.2)} {(0,50 \, m) (0,50 \, T)} = 0,39 \, A. \]

Значение

Это сильное магнитное поле создает значительную силу на длине провода, чтобы противодействовать его весу.

Пример \ (\ PageIndex {2} \): Расчет магнитной силы на токопроводящем проводе

Длинный жесткий провод, проложенный вдоль оси y , несет ток 5,0 А, текущий в положительном направлении y . (а) Если постоянное магнитное поле величиной 0.30 Тл направлено вдоль положительной оси x , какова сила магнитного поля на единицу длины провода? (b) Если постоянное магнитное поле 0,30 Тл направлено на 30 градусов от оси + x к оси + y , какова магнитная сила на единицу длины на проводе?

Стратегия

Магнитная сила, действующая на провод с током в магнитном поле, определяется выражением \ (\ vec {F} = I \ vec {l} \ times \ vec {B} \). Что касается части а, поскольку ток и магнитное поле в этой задаче перпендикулярны, мы можем упростить формулу, чтобы дать нам величину и найти направление через RHR-1.Угол θ составляет 90 градусов, что означает \ (sin \, \ theta = 1. \). Кроме того, длину можно разделить на левую часть, чтобы найти силу на единицу длины. Для части b текущая длина, умноженная на длину, записывается в обозначении единичного вектора, а также магнитное поле. После взятия перекрестного произведения направленность очевидна по результирующему единичному вектору.

Решение

  1. Начнем с общей формулы магнитной силы на проводе. Мы ищем силу на единицу длины, поэтому мы делим ее на длину, чтобы вывести ее в левую часть.Мы также устанавливаем \ (sin \, \ theta \). Следовательно, решением будет \ [F = IlB \, sin \, \ theta \] \ [\ frac {F} {l} = (5.0 \, A) (0.30 \, T) \] \ [\ frac {F} {l} = 1,5 \, Н / м. \] Направленность. Направьте пальцы в положительном направлении y и согните пальцы в положительном направлении x . Ваш большой палец будет указывать в направлении \ (- \ vec {k} \). Следовательно, с учетом направленности решение будет \ [\ frac {\ vec {F}} {l} = -1,5 \ vec {k} \, Н / м. \]
  2. Текущее значение, умноженное на длину, и магнитное поле записываются в виде единичного вектора.o) \ hat {i} \] \ [\ vec {F} / l = -1.30 \ hat {k} \, Н / м. \]

Значение

Это большое магнитное поле создает значительную силу на небольшой длине провода. По мере того, как угол магнитного поля становится более близким к току в проводе, на него действует меньшая сила, как видно из сравнения частей a и b.

Упражнение \ (\ PageIndex {1} \)

Прямой гибкий медный провод погружают в магнитное поле, направленное внутрь страницы.(а) Если ток в проводе течет в направлении + x , в каком направлении будет изгибаться провод? (b) В какую сторону изгибается провод, если ток течет в направлении — x -?

Решение

а. наклоняется вверх; б. наклоняется вниз

Пример \ (\ PageIndex {3} \): сила на круглом проводе

Круговая токовая петля радиусом R , по которой проходит ток I , размещена в плоскости xy . Постоянное однородное магнитное поле прорезает петлю параллельно оси y (рисунок \ (\ PageIndex {4} \)).Найдите магнитную силу на верхней половине петли, нижней половине петли и общую силу на петле.

Рисунок \ (\ PageIndex {4} \): петля из провода, по которой течет ток в магнитном поле.

Стратегия

Магнитная сила на верхнем контуре должна быть записана в терминах дифференциальной силы, действующей на каждый сегмент контура. Если мы интегрируем по каждому дифференциальному элементу, мы решаем общую силу на этом участке петли. Сила, действующая на нижнюю петлю, определяется аналогичным образом, а общая сила складывается из этих двух сил.

Решение

Дифференциальное усилие на произвольном отрезке проволоки, расположенном на верхнем кольце, составляет:

\ [dF = I B \, sin \, \ theta \, dl, \], где \ (\ theta \) — угол между направлением магнитного поля (+ y ) и отрезком провода. Дифференциальный сегмент расположен на том же радиусе, поэтому, используя формулу длины дуги, мы имеем:

\ [dl = Rd \ theta \]

\ [dF = IBR \, sin \, \ theta \, d \ theta. \]

Чтобы найти силу на отрезке, мы интегрируем по верхней половине круга от 0 до \ (\ pi \).0 sin \, \ theta \, d \ theta = IBR (-cos 0 + cos \ pi) = -2 IBR. \]

Чистая сила — это сумма этих сил, которая равна нулю.

Значение

Полная сила на любом замкнутом контуре в однородном магнитном поле равна нулю. Несмотря на то, что каждая часть петли имеет силу, действующую на нее, результирующая сила, действующая на систему, равна нулю. (Обратите внимание, что на петле есть чистый крутящий момент, который мы рассмотрим в следующем разделе.)

ЭЛЕКТРОМАГНИТНАЯ ИНДУКЦИЯ

ЭЛЕКТРОМАГНИТНАЯ ИНДУКЦИЯ

Рис 32.1 показан стержень из проводящего материала, перемещаемый с помощью скорости v в однородном магнитном поле B. Магнитная сила, действующая на свободную электрон в стержне будет направлен вверх и имеет величину

(32.1)

Рисунок 32.1. Движущийся проводник в магнитном поле. В результате действия магнитной силы электроны начнут накапливаются в верхней части стержня. Распределение заряда стержня будет поэтому измените, и верхушка стержня будет иметь избыток электронов (отрицательный заряд), а нижняя часть стержня будет иметь дефицит электронов (положительный заряд).Это распределение заряда будет создавать электрическое поле в стержень. Напряженность этого электрического поля будет увеличиваться до тех пор, пока электростатическая сила, создаваемая этим полем, равна по величине магнитная сила. В этот момент восходящий поток электронов остановится и

(32,2)

или

(32,3)

Индуцированное электрическое поле будет создавать разность потенциалов [Delta] V между концами стержня, равный

(32.4)

где L — длина стержня. Если концы стержня соединены с цепи, обеспечивающей возврат накопленного заряда, стержень будет источник ЭДС. Поскольку ЭДС связана с движением стержня через магнитное поле называется ЭДС движения . Уравнение (32.4) показывает, что величина ЭДС пропорциональна скорости v. на рисунке 32.1 мы видим, что vL — это площадь, пройденная стержнем на второй.Величина BvL — это магнитный поток, проходящий через стержень на второй. Таким образом

(32,5)

Хотя эта формула была получена для особого случая, показанного на рис. 32.1, она действительно в целом. Предназначен для перемещения стержней и проволоки произвольной формы. через произвольные магнитные поля.

Уравнение (32.5) связывает наведенную ЭДС со скоростью, с которой магнитный поток изменяется. В системе, показанной на рисунке 32.1 вложенный флюс изменяется из-за движения стержня. Вложенный магнитный поток также может быть изменяется при изменении напряженности приложенного магнитного поля. В обоих случаях результатом будет наведенная ЭДС. Связь между наведенной ЭДС и изменение магнитного потока известно как закон индукции Фарадея:

«Индуцированная ЭДС при движении или изменении математической траектории при постоянной или изменение магнитного поля равно скорости, с которой магнитный поток проходит через путь.»

Если рассматривать замкнутый путь, закон Фарадея можно сформулировать так:

«Индуцированная ЭДС вокруг замкнутого математического пути в магнитном поле равна к скорости изменения магнитного потока, перехваченного областью в пределах путь «

или

(32,6)

Знак минус в уравнении (32.6) указывает, насколько полярность наведенной ЭДС связаны со знаком потока и скоростью изменения потока.Знак поток фиксируется правилом правой руки:

«Согните пальцы правой руки в том направлении, в котором мы расчет ЭДС вокруг пути; тогда магнитный поток будет положительным, если линии магнитного поля указывают в направлении большого пальца, а отрицательные иначе. «

Пример: Задача 32.13

Металлический стержень длиной L и массой m скользит свободно, без трения, на двух параллельных металлических рельсах.Дорожки соединяются одним концом так, чтобы они и стержень образуют замкнутый контур (см. рисунок 32.2). У стержня есть сопротивление R, и гусеницы имеют незначительное сопротивление. Однородное магнитное поле перпендикулярно плоскости этого контура. Магнитное поле увеличивается при постоянной скорости дБ / дт. Первоначально магнитное поле имеет силу B 0 и стержень покоится на расстоянии x 0 от соединенный конец рельсов. Выразите ускорение стержня при этом момент в заданных количествах.

Рисунок 32.2. Проблема 32.13.

Магнитный поток [Phi], заключенный между стержнем и дорожками в момент времени t = 0 с, равен выдано

(32,7)

Магнитное поле увеличивается с постоянной скоростью, и, следовательно, закрытый магнитный поток также увеличивается:

(32,8)

Теперь для определения наведенной ЭДС можно использовать закон индукции Фарадея:

(32.9)

В результате наведенной ЭДС через стержень будет протекать ток с величина равна

(32.10)

Направление тока вдоль провода и, следовательно, перпендикулярно к магнитное поле. Сила, действующая на стержень со стороны магнитного поля, равна выдано

(32,11)

(см. главу 31). Комбинируя уравнения (32.10) и (32.11), получаем для силы на провод

(32.12)

Таким образом, ускорение стержня в момент времени t = 0 с равно

. (32,13) ​​

Пример: Задача 32.12

а) Длинный соленоид имеет 300 витков провода на метр и радиус 3,0 см. Если ток в проводе увеличивается со скоростью 50 А / с, то Скорость увеличения напряженности магнитного поля в соленоиде?

б) Соленоид окружен катушкой на 120 витков.Радиус этого катушка 6.0 см. Какая наведенная ЭДС будет генерироваться в этой катушке, пока ток в соленоиде увеличивается?

а) Магнитное поле в соленоиде обсуждалось в главе 31. Если соленоид имеет n витков на метр, и если I — ток через каждую катушку, то поле внутри соленоида равно

(32,14)

Следовательно,

(32,15)

В этой задаче n = 300 витков / метр и dI / dt = 50 А / с.Изменение в магнитное поле, таким образом, равно

(32,16)

Это уравнение показывает, что магнитное поле увеличивается со скоростью 0,019 Т / с.

б) Поскольку магнитное поле в соленоиде меняется, магнитный поток окруженная окружающей катушкой также изменится. Поток, заключенный в одинарная обмотка этой катушки

(32,17)

где r в = 3.0 см — радиус соленоида. Здесь у нас есть Предполагалось, что напряженность магнитного поля вне соленоида равна нулю. Суммарный поток, охватываемый внешними катушками, равен

. (32,18)

Скорость изменения магнитного потока из-за этого изменения магнитного поля равна выдано

(32,19)

В результате изменения тока в соленоиде будет наведена ЭДС во внешней катушке, со значением, равным

(32.20)

Если концы катушки соединены, ток будет течь через дирижер. Направление тока в катушке можно определить с помощью Закон Ленца , который гласит, что

«Индуцированные ЭДС всегда имеют такую ​​полярность, чтобы противодействовать изменение, которое их порождает «

Применим закон Ленца к задаче 32.12. Направление магнитного поля может быть определен с помощью правила правой руки и указывается вправо.Если ток в соленоиде увеличивается, магнитный поток также увеличивается. Электрический ток во внешней катушке будет течь в таком направлении, чтобы противодействовать этому изменению. Это означает, что ток в этой катушке будет течь против часовой стрелки ( поле, создаваемое индуцированным током, направлено противоположно полю генерируется большим соленоидом).

Стержень, движущийся в магнитном поле, будет иметь наведенную ЭДС в результате магнитная сила, действующая на свободные электроны.Индуцированная ЭДС будет пропорциональна линейной скорости v стержня. Если мы посмотрим на стержень из в системе отсчета, в которой стержень покоится, магнитная сила будет равна нулю. Однако все же должна быть наведенная ЭДС. Поскольку эта ЭДС не может быть генерируется магнитным полем, оно должно быть вызвано электрическим полем, которое существует в движущейся системе отсчета. Величина этого электрического поля должен быть таким, чтобы создавалась такая же наведенная ЭДС, что и в система отсчета, в которой движется стержень.Для этого требуется

(32.21)

Электрическое поле E ‘, существующее в системе отсчета движущегося стержня, равно называется индуцированным электрическим полем . ЭДС, возникающая между концы стержня

(32.22)

что эквивалентно уравнению (32.4). Если индуцированное электрическое поле имеет положение зависимой, то мы должны заменить уравнение (32.22) интегральным выражением

(32.23)

где интеграл простирается от одного конца стержня до другого конца стержня. стержень.

Разница между наведенным электрическим полем и электрическим полем генерируется статическим распределением заряда, состоит в том, что в первом случае поле не является консервативным и интеграл по путям по замкнутому пути равен

(32,24)

который не равен нулю, если магнитный поток зависит от времени.

Изменение тока в проводнике (например, в катушке) приводит к изменению магнитного поля. поле.Это зависящее от времени магнитное поле может вызвать ток за секунду. проводник, если он помещен в это поле. ЭДС, наведенная в эту секунду проводник, [эпсилон] 2 , будет зависеть от магнитного потока через этот проводник:

(32,25)

Поток [Phi] B1 зависит от напряженности магнитного поля. генерируется проводником 1 и, следовательно, пропорционален току I 1 через этот проводник:

(32.26)

Здесь постоянная L 21 зависит от размера двух катушек, от расстояние между ними и количество витков в каждой катушке. В Константа L 21 называется взаимной индуктивностью двух катушек. Используя эту константу, уравнение (32.25) можно переписать как

(32,27)

Единицей индуктивности является Генри (Гн), и из уравнения (32.27) мы заключаем, что

(32,28)

Когда магнитное поле, создаваемое катушкой, изменяется (из-за изменения ток) магнитный поток, заключенный в катушке, также изменится.Это изменение в потоке вызовет ЭДС в катушке, и поскольку ЭДС возникает из-за изменения ток через катушку называется ЭДС самоиндукции. В самоиндуцированная ЭДС равна

(32.29)

В уравнении (32.29) L называется собственной индуктивностью катушки. Самоиндуцированная ЭДС будет действовать в таком направлении, чтобы противодействовать изменению Текущий.

Пример: Задача 32.32

Длинный соленоид радиуса R имеет n витков на единицу длины.Циркуляр катушка из проволоки радиуса R ‘с n’ витками окружает соленоид. Что взаимная индукция? Имеет ли значение форма катушки с проволокой?

Предполагается, что поле внутри соленоида бесконечно длинное. соленоид и имеет силу равную

(32.30)

Поток, заключенный во внешнюю катушку, равен

(32.31)

Индуцированная ЭДС во внешней катушке равна

(32.32)

Таким образом, взаимная индуктивность L 12 равна

. (32,33)

Если через индуктор протекает постоянный ток, не зависящий от времени магнитный поле создано. Если вдруг источник тока отключится, изменение в заключенном магнитном потоке будет генерировать самоиндуцированную ЭДС, которая будет пытаться чтобы ток продолжал течь в первоначальном направлении. Электроэнергия доставляемая самоиндуцированной ЭДС изначально накапливалась в катушке индуктивности в форма магнитной энергии.Количество магнитной энергии, хранящейся в магнитном поле может быть определено путем расчета общей мощности, передаваемой мощностью источник для создания магнитного поля. Предположим, что после того, как батарея подключенный к катушке индуктивности, ток увеличивается со скоростью dI / dt. В самоиндуцированная ЭДС, создаваемая этим зависящим от времени током, равна

(32,34)

Ток должен обеспечивать дополнительную мощность, чтобы преодолеть эту самоиндуцированную ЭДС. В требуемая мощность будет зависеть от времени и равна

(32.35)

Работа, совершаемая током, сохраняется в индукторе в виде магнитной энергии. В изменение dU в магнитной энергии индуктора, таким образом, равно

(32,36)

Полная энергия, запасенная в магнитном поле индуктора, когда ток достигает своего окончательного значения, может быть получено интегрированием уравнения (32.36) между I = 0 и I = I f .

(32,37)

Для соленоида длиной l собственная индуктивность равна

. (32.38)

Таким образом, магнитная энергия, запасенная в соленоиде, равна

. (32,39)

где V — объем соленоида. Магнитная энергия может быть выражена в условия Б и В:

(32,40)

где B = u 0 n I — магнитное поле в соленоиде. Общая магнитная энергия индуктора теперь может быть выражена через плотность магнитной энергии u, которая определяется как

(32.41)

Магнитная энергия, запасенная в магнитном поле, равна плотности энергии время объем. Хотя мы вывели формулу для магнитной энергии плотности для частного случая очень длинного соленоида, его вывод действителен для любого произвольного магнитного поля.

Пример: Задача 32.46

Тороид квадратного сечения имеет внутренний радиус R 1 и внешний радиус R 2 .Тороид имеет N витков провода, несущего ток I; Предположим, что N очень велико.

а) Найдите плотность магнитной энергии как функцию радиуса.

б) Интегрируя плотность энергии, найдите полную магнитную энергию, хранящуюся в соленоид.

c) Выведите самоиндуктивность по формуле U = L . I 2 /2.

а) Примените закон Ампера, используя сферическую петлю Ампера с радиусом r

(32.42)

Ток, заключенный в амперовскую петлю, равен

. (32,43)

Используя закон Ампера, мы можем определить магнитное поле B:

(32,44)

Таким образом, плотность магнитной энергии равна

(32,45)

б) Пусть высота тороида равна h. Рассмотрим кусочек тороид показан на рисунке 32.3.

Рисунок 32.3. Сечение тороида задачи 32.46. Объем dV этого среза равен

(32,46)

Магнитная энергия, запасенная в этом сегменте, равна

(32,47)

Полная магнитная энергия, запасенная в тороиде, может быть получена путем интегрирования уравнение (32,47) относительно r между r = 1 и r = 2 :

(32,48)

в) Магнитная энергия, запасенная в индукторе с индуктивностью L, равна 0.5 л Я 2 . Сравнивая это с уравнением (32.48), мы заключаем, что собственная индуктивность L тороида равна

(32,49)

Цепь RL состоит из резистора и катушки индуктивности, включенных последовательно с аккумулятор (см. рисунок 32.4). Применяя к этому второму правилу Кричгофа по одноконтурной схеме получаем следующее дифференциальное уравнение

(32.50)

Рисунок 32.4. Схема RL. Это дифференциальное уравнение имеет решение

(32,51)

Это решение действительно, если батарея подключена при t = 0. Уравнение (32.51) показывает, что ток при t = 0 с равен 0 и неуклонно растет, достигая окончательное значение e / R при t = [бесконечность]. Постоянная времени цепи RL равна L / R. Если ток достиг постоянного значения и батарея внезапно разряжена. отключен, проводник может генерировать ток через резистор, который будет постепенно распадаться со временем.Если начальный ток равен [epsilon] / R, ток в момент времени t будет равен

(32,52)

Пример: Задача 32.54

Сколько джоулева тепла рассеивается током в уравнении (32,52) в резистор в интервале времени между t = 0 и t = [бесконечность]? Сравнить с начальная магнитная энергия в индукторе.

Ток через резистор указан в уравнении (32.51). Рассеиваемая мощность по этому ток в резисторе равен

(32.53)

Полная энергия, рассеиваемая этим током в резисторе между t = 0 и t = [бесконечность] равно

(32,54)

Магнитная энергия, запасенная в индукторе, равна

(32,55)

и мы заключаем, что вся магнитная энергия, запасенная в индукторе, рассеивается как джоулево тепло в резисторе.


Комментарии, вопросы и / или предложения отправляйте по электронной почте на адрес wolfs @ nsrl.rochester.edu и / или посетите домашнюю страницу Фрэнка Вольфса.

Объяснение правил Флеминга для левой и правой руки

Что такое правила Флеминга для левой и правой руки?

Всякий раз, когда проводник с током попадает в магнитное поле, на проводник будет действовать сила. Направление этой силы можно найти с помощью правила левой руки Флеминга (также известного как «правило левой руки Флеминга для двигателей»).

Точно так же, если проводник подвергается сильному воздействию магнитного поля, в этом проводнике будет индуцированный ток.Направление этой силы можно найти с помощью правила правой руки Флеминга.

В правилах как левой, так и правой руки Флеминга существует связь между магнитным полем, током и силой. Эта связь направленно определяется правилом левой руки Флеминга и правилом правой руки Флеминга соответственно.

Эти правила не определяют величину, а вместо этого показывают направление любого из трех параметров (магнитное поле, ток, сила), когда направление двух других параметров известно.

Правило левой руки Флеминга в основном применимо к электродвигателям, а правило правой руки Флеминга применимо в основном к электрическим генераторам.

Что такое правило левой руки Флеминга?

Обнаружено, что всякий раз, когда проводник с током помещается внутрь магнитного поля, на проводник действует сила в направлении, перпендикулярном как направлению тока, так и магнитного поля.

На рисунке ниже часть проводника длиной «L» помещена вертикально в однородное горизонтальное магнитное поле с напряженностью «H», создаваемое двумя магнитными полюсами N и S.Если через этот проводник протекает ток «I», величина силы, действующей на проводник, составляет:

Вытяните левую руку указательным, вторым и большим пальцами под прямым углом друг к другу. Если указательный палец представляет направление поля, а второй палец представляет направление тока, то большой палец указывает направление силы.

Когда ток течет по проводнику, вокруг него создается одно магнитное поле. Магнитное поле можно представить, рассматривая количество замкнутых магнитных силовых линий вокруг проводника.

Направление магнитных силовых линий может быть определено правилом штопора Максвелла или правилом правостороннего захвата.

Согласно этим правилам, направление магнитных силовых линий (или силовых линий) — по часовой стрелке, если ток течет от наблюдателя, то есть если направление тока через проводник направлено внутрь от плоскости отсчета, как показано на рисунке.


Теперь, если горизонтальное магнитное поле приложено извне к проводнику, эти два магнитных поля i.е. поле вокруг проводника из-за проходящего через него тока и приложенного извне поля будут взаимодействовать друг с другом.

На рисунке мы видим, что силовые линии внешнего магнитного поля проходят от северного к южному полюсу, то есть слева направо.

Магнитные силовые линии внешнего магнитного поля и магнитные силовые линии, обусловленные током в проводнике, находятся в одном направлении над проводником и в противоположном направлении под проводником.

Следовательно, будет большее количество сонаправленных магнитных силовых линий над проводником, чем под проводником.

Следовательно, в небольшом пространстве над проводником будет большая концентрация магнитных силовых линий. Поскольку магнитные силовые линии больше не являются прямыми линиями, они находятся под натяжением, как натянутые резиновые ленты.

В результате возникнет сила, которая будет стремиться переместить проводник из более концентрированного магнитного поля в менее концентрированное магнитное поле, то есть из текущего положения вниз.

Теперь, если вы заметите направление тока, силы и магнитного поля в приведенном выше объяснении, вы обнаружите, что направления соответствуют правилу левой руки Флеминга.

Что такое правило правой руки Флеминга?

Согласно закону электромагнитной индукции Фарадея, всякий раз, когда проводник движется внутри магнитного поля, в нем будет индуцированный ток. Если этот проводник будет принудительно перемещен внутри магнитного поля, возникнет связь между направлением приложенной силы, магнитным полем и током.

Это соотношение между этими тремя направлениями определяется Правилом правой руки Флеминга.

Это правило гласит: «Вытяните правую руку первым, вторым и большим пальцами под прямым углом друг к другу. Если указательный палец представляет направление силовой линии, большой палец указывает в направлении движения или приложенной силы, то второй палец указывает в направлении индуцированного тока ».

Кто изобрел правила для большого пальца левой и правой руки?

Правила для большого пальца левой и правой руки были основаны Джоном Амброузом Флемингом в конце 19 -х годов века.

Джон открыл оба этих правила и назвал их в честь себя. Правила теперь хорошо известны как правило левой и правой руки Флеминга .

Джон Амброуз Флеминг

Закон Фарадея, закон Ампера, закон Ленца и сила Лоренца

Работа электродвигателей регулируется различными законами электричества и магнетизма, включая закон индукции Фарадея, закон оборота Ампера, закон Ленца, и сила Лоренца. Первые два — закон Фарадея и закон Ампера — включены в уравнения Максвелла.Вместе с законом Ленца и силой Лоренца эти принципы составляют основу электромагнетизма.


Закон индукции Фарадея

Закон индукции Фарадея — это фундаментальный закон, по которому работают электродвигатели. Майкл Фарадей приписывают открытие индукции в 1831 году, но Джеймс Клерк Максвелл описал ее математически и использовал в качестве основы своей количественной электромагнитной теории в 1860-х годах.


Индуктивность — это свойство устройства, которое показывает, насколько эффективно оно индуцирует ЭДС в другом устройстве (или на самом себе).


Закон Фарадея обычно гласит, что в замкнутой катушке (петле) провода изменение магнитной среды катушки вызывает в катушке напряжение или ЭДС (электродвижущую силу).

Изменение магнитной среды может быть вызвано изменением напряженности магнитного поля, перемещением магнита по направлению к катушке или от нее, перемещением катушки в магнитное поле или из него или вращением катушки в поле.

Индуцированная ЭДС равна отрицательной скорости изменения магнитного потока, умноженной на количество витков в катушке:

Где:

E = ЭДС (В)

N = количество витков в катушке

Φ = магнитный поток (Вебер, Вт)

t = время (с)


Обратите внимание, что магнитный поток равен среднему магнитному полю, B, (тесла или Вт / м 2 ), умноженному на площадь перпендикуляра катушки, которая проникает в магнитное поле, A (м 2 ).


Закон Ленца

Закон Ленца демонстрирует причину отрицательного знака в законе индукции Фарадея. Другими словами, закон Ленца объясняет , почему ЭДС, генерируемая в соответствии с законом Фарадея, отрицательна.

Обычный способ сформулировать закон Ленца: «Когда ЭДС генерируется изменением магнитного потока, полярность индуцированной ЭДС такова, что она генерирует ток, магнитное поле которого направлено в направлении, противоположном изменению, которое его вызвало. (исходное магнитное поле).То есть индуцированное магнитное поле всегда поддерживает постоянный магнитный поток.

Когда магнитный поток изменяется (ΔB), магнитное поле наведенной ЭДС (B Induced ) работает, чтобы противодействовать изменению.
Изображение предоставлено: К. Р. Нейв, Государственный университет Джорджии

Закон Ленца аналогичен третьему закону Ньютона в механике, который гласит, что для каждого действия существует равное и противоположное противодействие.


Сила Лоренца

Существуют разногласия по поводу того, была ли сила Лоренца первоначально получена Джеймсом Клерком Максвеллом или Оливером Хевисайдом, но, как правило, заслуги передаются Хевисайду.Хендрик Лоренц вывел современную форму уравнения в 1891 году.

Сила Лоренца — это сила, которую частица испытывает из-за электрического и магнитного полей. Электрические поля действуют на частицу независимо от того, движется она или нет, в то время как магнитные поля действуют только тогда, когда частица находится в движении. Комбинация сил электрического и магнитного полей определяется как:

Что упрощается до:

Где:

F ​​= сила (Н)

q = частица заряда (кулон, Кл)

E = электрическое поле (N / C)

v = скорость перпендикулярно магнитному полю (м / с)

B = магнитное поле (тесла, Тл)

Поскольку ток — это, по сути, поток движущихся заряженных частиц, он также испытывает силу, обусловленную магнитным полем.В случае тока в магнитном поле уравнение силы Лоренца принимает вид:

Где:

I = ток (А)

l = длина провода через поле (м)

Направление силы Лоренца определяется с использованием правила правой руки: направьте большой палец в направлении тока, первый палец — в направлении магнитного поля, а второй (средний) палец — в направлении тока. сила.


Окружной закон Ампера

Несмотря на свое название, круговой закон Ампера был выведен не Андре-Мари Ампера, а Джеймсом Клерком Максвеллом в 1860 году и является одним из уравнений Максвелла электромагнетизма. (Ампер сформулировал закон силы Ампера, который описывает силу притяжения или отталкивания между двумя токоведущими проводами.)

Магнитное поле действует на прямой провод, по которому течет ток. По закону движения Ампера напряженность магнитного поля может быть определена по формуле:

Где:

B = магнитное поле (Тл)

μ 0 = магнитная проницаемость воздуха, Т-м / А

I = ток (А)

r = расстояние от провода (м)

Когда провод представляет собой петлю, магнитное поле вызывает силу в одном направлении на одной стороне петли и в противоположном направлении на другой стороне петли.Это создает крутящий момент, который заставляет катушку вращаться. Обратите внимание, что при подаче постоянного тока катушка будет колебаться вперед и назад, но не будет совершать полных оборотов — это причина, по которой в двигателях постоянного тока используются коммутаторы. Двигатели, работающие на переменном токе (двигатели переменного тока), не имеют этой проблемы.

Изображение предоставлено: TutorVista.com

Что такое сила Ампера? — Естественные науки 2021

Сила Ампера — это сила, с которой магнитное поле действует на проводник с протекающим в нем током.Его направление можно определить как по правилу левой руки, так и по правилу по часовой стрелке.

Инструкция по эксплуатации

1

Если металлический проводник с током поместить в магнитное поле, то сила этого поля, сила Ампера, будет действовать на него. Ток в металле — это направленное движение множества электронов, на каждый из которых действует сила Лоренца. Силы, действующие на свободные электроны, имеют одинаковую величину и одинаковое направление. Сложив вместе, они дают результирующую силу Ампера.

2

Сила получила свое название в честь французского физика и естествоиспытателя Андре Мари Ампера, который в 1820 году экспериментально исследовал действие магнитного поля на проводник с током. Изменяя форму проводников, а также их расположение в магнитном поле, Ампер определил силу, действующую на отдельные участки проводника.

3

Модуль силы Ампера пропорционален длине проводника, силе тока в нем и модулю индукции магнитного поля.Это также зависит от ориентации проводника в магнитном поле, другими словами, от угла, который формирует направление тока по отношению к вектору индукции магнитного поля.

4

Если индукция во всех точках проводника одинакова и магнитное поле однородно, то модуль силы тока равен произведению силы тока в проводнике, модуля магнитной индукции, в котором он расположен, и длины этого проводника и синус угла между направлениями тока и вектора индукции магнитного поля.Эта формула верна для проводника любой длины, если при этом он полностью находится в однородном магнитном поле.

5

Чтобы узнать направление силы Ампера, вы можете применить правило левой руки: если вы положите левую руку так, чтобы четыре ее пальца указывали направление тока, а линии поля входили в ладонь вашей руке направление силы Ампера покажет большой палец, согнутый на 90 °.

6

Поскольку произведение модуля вектора индукции магнитного поля на синус угла является модулем составляющей вектора индукции, которая направлена ​​перпендикулярно проводнику с током, ориентация ладони может быть определена этим составная часть.Перпендикулярный компонент к поверхности проводника должен одновременно входить в раскрытую ладонь левой руки.

7

Есть еще один способ определения направления силы Ампера, он называется правилом по часовой стрелке. Сила Ампера направлена ​​против часовой стрелки в том направлении, откуда наблюдается кратчайшее текущее вращение к полю.

Author: alexxlab

Добавить комментарий

Ваш адрес email не будет опубликован. Обязательные поля помечены *